FREE SAMPLE TESTS

Air Brake

Sample Test A

1: What are the three braking systems?

  1. The parking brake and emergency brake and thermal system
  2. The service brake and parking brake and emergency system
  3. The service brake and emergency and the inverse system

2: What is used to make the brakes work?

  1. Compressed Oxygen
  2. Compressed Nitrogen
  3. Compressed air

3: What does the air compressor do?

  1. Pumps air into the air storage tanks
  2. Keeps the tires inflated to proper pressure
  3. Comes on only if you have to make an emergency stop

4: What does the air compressor governor control?

  1. It controls emergency warning systems
  2. It controls when the compressor will pump air into the air storage tanks
  3. It controls how the compressor will pump air into the air lines

5: How much compressed air must the storage tanks hold?

  1. At least one hundred and twenty pounds of air
  2. A maximum of one atmosphere
  3. Enough air to allow brakes to be used when the air compressor stops working

6: If oil and water collect in the air tanks what can happen to the brakes?

  1. The brakes can fail
  2. Brake linings will automatically slip loose
  3. The brakes will work better

7: No automatic tank drains - when should you drain the air tanks?

  1. Once a week
  2. Once a month
  3. Every day

8: Where will you find the drain valve for each air tank?

  1. In the bottom of the tank
  2. At the top of the tank
  3. It is usually found above the tank

9: What are the two types of air tank drains?

  1. Manually operated. . . Emergency
  2. Automatic . . . Emergency
  3. Manually operated. . . Automatic

10: What is a purpose of an alcohol evaporator?

  1. It takes alcohol from the air system
  2. It reduces risk of ice in brake valves and other parts in cold weather
  3. It is designed to save fuel in warm weather

Question Your Answer Correct Answer

Sample Test B

1: How often should you check the alcohol evaporator in cold weather?

  1. Check the alcohol container and fill up as necessary once each week
  2. Check the alcohol container once each fall before cold weather starts
  3. Check the alcohol container and fill up as necessary every day

2: What is a purpose of the safety release valve in the first tank?

  1. It protects the tank and the rest of the system from too much pressure
  2. It protects all of the system from dirt and oil
  3. It helps build up air pressure quickly without running the engine

3: What are two other names for the brake pedal?

  1. It can be called the foot valve or the relay valve
  2. It can be called the foot valve or treadle valve
  3. It can be called the relay valve or air compressor valve

4: What can happen if the brake pedal is pressed and released too often?

  1. The brakes will be cooled down as soon as all the hot air is removed
  2. Air pressure can build up until the brake pressure gauge is broken
  3. Air can be let out of the system faster than the compressor can replace

5: Where are foundation brakes found?

  1. At each wheel
  2. On every other axle
  3. On the drive wheels only

6: Where are brake shoes and linings located?

  1. Directly beneath the foot valve
  2. Inside each brake drum
  3. On the outside of certain brake drums

7: What happens when a brake lining is pushed against the inside of the drum?

  1. The brakes will always squeal
  2. This causes friction which slows the vehicle and creates heat
  3. This causes heat which automatically causes the brakes to lock up

8: On S-Cam brakes what happens to the air when you push the brake pedal?

  1. Air pressure forces out the push rod and moves the slack adjuster
  2. Air pressure works only with wedge brakes and not with S-Cam brakes
  3. Air pressure is converted into hydraulic pressure

9: Which direction does the S-Cam force brake shoes when brakes are applied?

  1. It presses them against the outside of the brake drum
  2. It presses them against the inside of the brake drum
  3. It presses them around the brake drum

10: If the air compressor begins to leak what keeps air in the tanks?

  1. The slack adjusters
  2. The alcohol evaporator
  3. The one-way check valve

Question Your Answer Correct Answer

Sample Test C

1: What is one kind of gauge that is required for vehicles with air brakes?

  1. An air temperature gauge
  2. An air pressure gauge
  3. An oil pressure gauge

2: What kind of warning signal is required on vehicles with air brakes?

  1. A high air pressure warning signal
  2. A low air pressure warning signal
  3. A changing air pressure warning signal

3: What are the three types of low air pressure warning devices?

  1. A siren . . . or a horn . . . or 4-way flashers
  2. A red light . . . or a buzzer . . . or a wigwag
  3. An air horn . . . or an amber light . . . or a whistle

4: What is used to turn on the stop lights in an air brakes system?

  1. An electric switch that works by air pressure
  2. A computer
  3. A motion sensor in the wheels

5: In normal driving parking and emergency brakes are usually held back by?

  1. Electric relay switches
  2. Wheel chocks
  3. Air pressure

6: The effectiveness of the spring brakes depends on the adjustment of?

  1. Emergency brakes
  2. Service brakes
  3. Front wheel brakes

7: Why do some buses have a separate air tank to release spring brakes?

  1. So you can move the vehicle in an emergency
  2. To simplify servicing the air brake system
  3. So that the brakes will release quicker

8: What is a dual air brake system?

  1. Two separate air brake systems with a single set of brake controls
  2. One air brake system with two sets of brake controls
  3. Two separate air brake systems with two sets of brake controls

9: What should you look for in checking an air compressor belt?

  1. Check for belts that are the wrong color
  2. Check for belts that are not made of rubber
  3. Check belts for excessive wear and cracks and tightness

10: What should you do before checking free play in manual slack adjusters?

  1. Park in a secure location and remove all wheel chocks
  2. Park on level ground so that the wheels will not have to be chocked
  3. Park on level ground and chock the wheels and release parking brakes

Question Your Answer Correct Answer

Sample Test D

1: Combination vehicle: brakes released. The maximum air released?

  1. One pound per square inch
  2. Less than two pounds per square inch
  3. Less than three pounds per square inch

2: Single vehicle: brakes released. Maximum air leakage in one minute?

  1. One pound per square inch
  2. Less than two pounds per square inch
  3. Less than three pounds per square inch

3: After applying brakes fully, maximum air loss in one minute - single vehicles?

  1. Two pounds per square inch
  2. Three pounds per square inch
  3. Four pounds per square inch

4: After applying brakes fully max. air loss in one minute - combination vehicles?

  1. Two pounds per square inch
  2. Three pounds per square inch
  3. Four pounds per square inch

5: When should the air governor cut out the air compressor?

  1. At about the manufacturer specified air pressure
  2. At about ten pounds per square inch
  3. At about one hundred and ten pounds per square inch

6: When making a normal stop when should you push in the clutch?

  1. Experienced drivers do not use the clutch
  2. Do not push in the clutch until there is vibration
  3. Do not push the clutch in until the engine R.P.M.'s are down close to idle

7: When making a very quick stop you should brake so that you?

  1. Can turn quickly to get out the way of hazards
  2. Skid to maximize stopping distance
  3. Stay in a straight line and can steer

8: What is another name for controlled braking?

  1. Steady braking
  2. Hard braking
  3. Squeeze braking

9: What is stab braking?

  1. Putting the brakes on hard without locking the wheels or turning
  2. Pressing the brake pedal hard and releasing brakes when wheels lock up
  3. Applying a constant pressure to the brakes

10: Why does air braking take more time than hydraulic braking?

  1. Because air brakes are generally used on longer vehicles
  2. Because it takes time for air to flow through the lines to the brakes
  3. Because air moves through the lines much slower than oil does

Question Your Answer Correct Answer

Sample Test E

1: With dual air systems, the air pressure should build:

  1. From 85 to 100 psi in 45 seconds
  2. From 70 to 100 psi in 60 seconds
  3. From 85 to 150 psi in 45 seconds
  4. From 70 to 150 psi in 60 seconds

2: A fully charged air system typically has ____ psi:

  1. 100
  2. 125
  3. 150
  4. 175

3: To test air leakage rate you should:

  1. Leave the engine running and release the service brake
  2. Turn off the engine and release the service brake
  3. Charge the air system and leave the engine running
  4. Depress the brake pedal and monitor the air pressure gauge

4: A combination vehicle air brake system should not lose air faster than:

  1. 2 psi per minute
  2. 3 psi per 30 seconds
  3. 3 psi per minute
  4. 4 psi per minute

5: To check the air compressor governor "cut in" and "cut out" pressures:

  1. Run the engine at a fast idle
  2. Step on and off the brake to reduce tank pressure
  3. Monitor the air pressure gauge
  4. All of the above

6: To test your service brakes you should:

  1. Stop the vehicle and put the parking brake on
  2. Release the parking brake, move forward slowly and depress brakes
  3. Put parking brake on and release air pressure
  4. Depress brakes when traveling at highway speeds

7: Stab braking occurs when you:

  1. Press on the brake pedal as hard as you can
  2. Release the brakes when the wheels lock up
  3. Apply the brakes when the wheels start rolling
  4. All of the above

8: Air brakes take longer to stop than hydraulic brakes because:

  1. The reaction time is longer for the driver
  2. It takes longer for the air to flow through the lines
  3. The linings get hotter more quickly
  4. None of the above

9: The proper way to go down long grades is to:

  1. Use a low gear and push hard on the brakes
  2. Let up on the brakes from time to time to cool them
  3. Use a low gear and apply light pressure on the brakes
  4. Brake on and off down the hill

10: You should use the parking brake:

  1. Only on steep hills
  2. On level surfaces
  3. Any time you park
  4. When the brakes are hot

Question Your Answer Correct Answer

Sample Test F

1: When you apply pressure to the brake pedal, you are:

  1. Pushing against a spring and against air pressure to the brakes
  2. Releasing air pressure to the brakes
  3. Reducing air in the tanks
  4. Activating the safety valve

2: Brake drums:

  1. Are bolted to the wheels
  2. Are located at each end of the vehicle's axles
  3. Hold the brake shoes and linings
  4. All of the above

3: Which type of brake is the most common in commercial vehicles?

  1. Wedge brakes
  2. Disk brakes
  3. S Cam brakes
  4. Drum brakes

4: An air pressure gauge:

  1. Shows how much pressure you are applying to the brakes
  2. Is connected to the generator
  3. Releases air when you accelerate
  4. Prevents brake failure

5: All vehicles with air bakes are required to have a low pressure warning signal. This could be:

  1. A red light
  2. A wig wag mechanical arm
  3. A buzzer
  4. Any one of the above

6: When the low pressure warning is activated, you should:

  1. Wait for the air brakes to come on automatically
  2. Bring the vehicle to a safe stop while you still control the brakes
  3. Monitor your air pressure gauge and maintain highway speeds
  4. Pump the brake pedal hard to stop

7: A duel air brake system:

  1. Has two air brake systems and one set of controls
  2. Takes less time to build air pressure
  3. Has one set of air tanks and hoses
  4. Must have two warning alarms

8: An emergency brake:

  1. Is required on commercial vehicles
  2. Must be held on by mechanical force
  3. Could be activated by the loss of air pressure
  4. All of the above

9: To check the slack adjusters on S Cam brakes, you should:

  1. Leave the parking brakes on and pull on the slack adjuster
  2. Wear gloves and pull hard on each slack adjuster
  3. Make sure the slack adjusters move more than one inch each way
  4. Drain the air from the system and pull on the slack adjusters

10: To test the low pressure warning signal, you should do all BUT the following:

  1. Shut the engine off
  2. Turn the electrical power on
  3. Ask a friend to listen for the signal
  4. Step on and off the brake pedal to reduce air pressure

Question Your Answer Correct Answer

Sample Test G

1: The air compressor is connected to the engine through:

  1. Gears
  2. V Belts
  3. Hoses
  4. Both A and B

2: The Air Compressor Governor controls:

  1. When the brakes should be applied
  2. When the air compressor pumps air into the storage tanks
  3. When the warning light is activated
  4. When the air is released from the tank

3: Compressed air is stored:

  1. In the hoses
  2. In the engine compartment
  3. In the fuel tank
  4. In the storage tank

4: The Governor stops pumping air when it reaches the "cut out" level, or around:

  1. 125 psi
  2. 200 psi
  3. 225 psi
  4. 175 psi

5: The Governor allows the air to start pumping again when it reaches the cut in level, or around:

  1. 200 psi
  2. 100 psi
  3. 150 psi
  4. 125 psi

6: An air tank is equipped with a drain valve to remove:

  1. Air and water
  2. Oil and air
  3. Water and oil
  4. Dirt and oil

7: Some air brake systems have an alcohol evaporator which helps:

  1. Reduce the risk of ice in the air brake valves during cold weather
  2. Remove oil from the air lines
  3. Reduce the risk of water in the air brake valves in warm weather
  4. Remove fuel from the air lines

8: The air tank should be drained how often?

  1. Once a week
  2. Once a day
  3. Every four hours
  4. Every two or three days

9: The safety valve in the air compressor tank is set to open at:

  1. 150 psi
  2. 200 psi
  3. 100 psi
  4. 175 psi

10: When the safety valve releases air it means that:

  1. The pressure has exceeded 150 psi
  2. Too much air is in the air compressor
  3. Something is wrong, and you should have it checked by a mechanic
  4. All of the above

Question Your Answer Correct Answer

Bus and Coach Endorsement

Sample Test A

1: In addition to checking for spare electrical fuses, three red reflective triangles, and a properly charged and rated fire extinguisher, school bus drivers must also inspect the following emergency equipment:

  1. three red burning flares, safety belts in all seats.
  2. three red burning flares, a nine-item first-aid kit.
  3. three flares of any type and isopryl alcohol.
  4. none of the above.

2: A school bus driver must also check the alternately flashing amber lights indicator, if equipped, the alternately flashing red lights indicator and the strobe light indicator, if equipped.

  1. TRUE
  2. FALSE
  3. could be either
  4. none of the above.

3: School bus drivers must also check the following external lights and reflectors:

  1. strobe light, if equipped, stop arm light, if equipped.
  2. alternately flashing amber lights, if equipped.
  3. alternately flashing red lights.
  4. none of the above.

4: If equipped, check the stop arm to see that it is mounted securely to the left front window of the school bus.

  1. TRUE
  2. FALSE
  3. could be either.
  4. none of the above.

5: Check that the entry door is not damaged and:

  1. operates smoothly and closes securely from the inside.
  2. operates smoothly and closes securely from the outside
  3. operates smoothly and closes securely from a remote location.
  4. none of the above.

6: Should the hand rails and stop light be checked during a pre-trip inspection?

  1. Yes
  2. No
  3. could be either.
  4. none of the above.

7: On a pre-trip inspection for a bus, the passenger/entry should be checked for:

  1. door operation, hand and foot rails and handicap lift
  2. door operation, hand rails, entry step condition and brake condition.
  3. door operation, hand rails, entry step condition and handicap lift if equipped
  4. none of the above.

8: When checking a handicap lift the things that you should look for are:

  1. leaking, damaged or missing wheel chairs
  2. leaking, damaged, or missing parts and explain how lift should be checked for correct operation. Lift must be fully retracted and latched.
  3. leaking, damaged, or missing parts and explain how lift should be checked for correct operation. Lift must be 25 percent retracted and latched.
  4. none of the above.

9: Make sure that all emergency exits are not damaged, operate smoothly and close securely from the outside

  1. TRUE
  2. FALSE
  3. could be either.
  4. none of the above.

10: Check all emergency exits and show that:

  1. they are not damamged, operate smoothly, and close securely from the inside and the warning devices are working.
  2. they are damaged, operate smoothly and close securely and exit warning devices are working
  3. all devices inside and outside of bus are operating in unison
  4. none of the above.

Question Your Answer Correct Answer

Sample Test B

1: Name some things to check in the interior of a bus during the pre-trip inspection.

  1. Parking brake and steering mechanism
  2. Lights, reflectors, and horn
  3. Tires and windshield wipers
  4. All of the above

2: As you check the outside of the bus, you should close any open emergency exits or open access panels.

  1. TRUE
  2. FALSE

3: When checking the interior of a bus before driving it which of the following parts of the bus must be in safe working condition?

  1. Signaling devices (emergency buzzer)
  2. Handrails, emergency exit handles
  3. Air Conditioners and heaters
  4. A and B

4: Are bus drivers required to wear seatbelts?

  1. Yes
  2. No

5: Is it alright to leave carry on baggage in a doorway or aisle?

  1. Yes
  2. No

6: Most hazardous materials cannot be carried on a bus, but some hazardous materials are allowed.

  1. TRUE
  2. FALSE

7: What are some hazardous materials you can transport by bus?

  1. Poison, tear gas, and irritating materials
  2. Small arms ammunition, hospital supplies, and drugs
  3. Explosives and radioactive materials
  4. None of the above

8: In buses designed to allow standing you can stand anywhere you want.

  1. TRUE
  2. FALSE

9: What is a standee line?

  1. A strip around the outside of the bus
  2. A two inch line on the floor to show riders where not to stand
  3. A two inch line between the seats
  4. Any of the above

10: Does it matter where you make a disruptive passenger get off the bus?

  1. Yes
  2. No

Question Your Answer Correct Answer

Sample Test C

1: A school bus is "every motor vehicle used for the transportation of children to or from school or school activities."

  1. TRUE
  2. FALSE

2: It doesn't matter if school bus drivers know current laws and regulatons and keep abreast of any changes

  1. TRUE
  2. FALSE

3: School bus drivers must have a commercial driver's license and:

  1. passenger and combination vehicle endorsements
  2. school bus and passenger endorsements
  3. air brake and tank endorsements
  4. none of the above

4: Maximum time in transit: No pupil shall be allowed to remain in transit to and from school for:

  1. more than one hour round trip
  2. more than two hours round trip
  3. more than three hours round trip.
  4. more than four hours round trip.

5: Can a school bus transport more pupils than indicated by the manufacturer's rated capacity for the bus?

  1. Yes
  2. No

6: Inspection of buses are made one or more times a year in order to determine wherher the school bus can be used to safely transport school children.

  1. TRUE
  2. FALSE

7: Each state board of education is charged with the primary responsibility of rules and regulations regarding pupil transportation.

  1. TRUE
  2. FALSE

8: A school bus driver doesn't have to wear a seat belt at all times

  1. TRUE
  2. FALSE

9: Only some school bus drivers are required to participate in In-service training programs.

  1. TRUE
  2. FALSE

10: Who makes the final decision to determine when weather conditions make the roads unsafe to travel?

  1. The School Administrator
  2. The School Teacher
  3. The School Bus Driver
  4. Any of the above

Question Your Answer Correct Answer

Sample Test D

1: Should the hand rails and stop light be checked during a pre-trip inspection?

  1. Yes
  2. No

2: On a pre-trip inspection for a bus, the passenger/entry should be checked for:

  1. door operation, hand and foot rails and handicap lift.
  2. door operation, hand rails, entry step condition and brake condition
  3. door operation, hand rails, entry step condition and handicap lift if equipped
  4. door operation, hand rails, entry step condition, handicap lift if equipped and emergency lighting

3: When checking a handicap lift the things that you should look for are:

  1. leaking, damaged or missing wheel chairs
  2. leaking, damaged, or missing parts and explain how lift should be checked for correct operation. Lift must be fully retracted and latched.
  3. leaking, damaged, or missing parts and explain how lift should be checked for correct operation. Lift must be 25 percent retracted and latched
  4. wheel chair chocks

4: Make sure that all emergency exits are not damaged, operate smoothly and close securely from the outside.

  1. TRUE
  2. FALSE

5: Check all emergency exits and show that:

  1. they are not damaged, operate smoothly, and close securely from the inside and the warning devices are working
  2. they are damaged, operate smoothly and close securely and exit warning devices are working
  3. all devices inside and outside of bus are operating in unison
  4. the reflective tape is clean

6: During the pre-trip inspection you should check for broken seat frames and check that seat frames are firmly attached to the floor.

  1. TRUE
  2. FALSE

7: Should you check passenger exit mirrors?

  1. Yes
  2. No

8: During the external inspection of a Coach or Transit Bus a check should be made to see that:

  1. the vehicle is sitting level (side to side) and if air-equipped check for water leakage.
  2. the vehicle is not sitting level and if air equipped check for audible air leaks from the suspension system.
  3. the vehicle is sitting level (front and rear) and if air-equipped, check for audible air leaks from the suspension system
  4. none of the above

9: A compartment check includes:

  1. checking that baggage and all other exterior compartment doors are not damaged, operate properly and latch securely.
  2. checking that all compartment doors are painted in coordinating colors
  3. checking that everyone has brought baggage
  4. all of the above.

10: A battery/box check is important. The battery must be secure, connections must be tight and cell caps must be present. In addition these items should be checked:

  1. battery connections should show signs of excessive wear
  2. battery connections should not show signs of excessive corrosion, but the battery box and cover or door is unimportant.
  3. battery connections should not show signs of excessive corrosion and the battery box and cover or door should not be damaged and should be secure
  4. that the mechanic initialed any work performed on the battery

Question Your Answer Correct Answer

Sample Test E

1: How far from a railroad crossing should you stop?

  1. Between 10 and 40 feet
  2. Between 15 and 50 feet
  3. Between 20 and 60 feet
  4. Between 25 and 75 feet

2: When must you stop before crossing a drawbridge?

  1. Stop at least 30 feet before the draw of the bridge
  2. Stop at least 40 feet before the draw of the bridge.
  3. Stop at least 50 feet before the draw of the bridge
  4. None of the above

3: What are"prohibited practices" while operating a bus?

  1. Avoid fueling with riders on board.
  2. Don't talk to riders while driving
  3. Don't tow or push a disabled bus with riders aboard
  4. All of the above

4: Urban mass transit coaches may have a brake and accelerator interlock shystem. The rear door of a transit bus has to be open to put on the parking brake.

  1. TRUE
  2. FALSE

5: If you work as an interstate carrier, you must complete a written inspection report for each bus driver

  1. TRUE
  2. FALSE

6: Are recapped or regrooved tires allowed on buses?

  1. Yes, on all wheels
  2. Yes, on all wheels except the front wheels.
  3. No, recapped or regrooved tires are not allowed on buses
  4. As a spare only

7: Before driving who is responsible for inspecting emergency equipment?

  1. The shop
  2. The dispatcher
  3. The driver
  4. All of the above

8: What are the shape of hazardous material labels?

  1. Round
  2. Rectangular
  3. Diamond-shaped
  4. Oval

9: Which poor weather condition can cause the most dangerous driving?

  1. Ice
  2. Rain
  3. Snow
  4. Dark Clouds

10: If you have to swerve quickly to avoid an accident you want to know:

  1. If all of your passengers are wearing seat belts
  2. Where other vehicles are around your bus
  3. Where your passengers are seated.
  4. If your emergency flashers work properly

Question Your Answer Correct Answer

Sample Test F

1: During the pre-trip inspection you should check for broken seat frames and check that seat frames are firmly attached to the floor

  1. TRUE
  2. FALSE

2: Should you check passenger exit mirrors?

  1. TRUE
  2. FALSE

3: During the external inspection of a Coach or Transit Bus a check should be made to see that:

  1. the vehicle is sitting level (side to side) and if air-equipped check for water leakage
  2. the vehicle is not sitting level and if air equipped check for audible air leaks from the suspension system
  3. the vehicle is sitting level (front and rear) and if air-equipped, check for audible air leaks from the suspension system.
  4. none of the above

4: A compartment check includes:

  1. checking that baggage and all other exterior compartment doors are not damaged, operate properly and latch securely
  2. checking that all compartment doors are painted in coordinating colors
  3. checking that everyone has brought baggage
  4. all of the above.

5: A battery/box check is important. The battery must be secure, connections must be tight and cell caps must be present. In addition these items should be checked:

  1. battery connections should show signs of excessive wear
  2. battery connections should not show signs of excessive corrosion, but the battery box and cover or door is unimportant.
  3. battery connections should not show signs of excessive corrosion and the battery box and cover or door should not be damaged and should be secure
  4. that the mechanic initialed any work performed on the battery

6: What is one of the more dangerous procedures a school bus driver must undertake?

  1. Driving in traffic
  2. Loading and unloading
  3. Driving in the rain
  4. Training replacement drivers

7: When approaching a school bus stop you should activate your overhead amber lights how far from the bus stop?

  1. 100 ft
  2. 200 ft
  3. 300 ft
  4. anytime before coming to a stop.

8: Immediately after stopping you should:

  1. Open entrance door slighly to activate the stop arms and overhead red warning lights
  2. Tell the children to stand back until you are ready for them to load.
  3. Get the children onto the bus as quickly as possible
  4. Turn the engine off and open the door

9: Always unload on the right outside lane if on a multi-lane road.

  1. TRUE
  2. FALSE

10: Pupils must walk at least how many feet in front of a school bus to cross the road?

  1. 8 ft
  2. 10 ft.
  3. 12 ft
  4. 15 ft

Question Your Answer Correct Answer

Cargo Tank Combination Vehicle

Cargo Test A

1: A 'tank vehicle' includes?

  1. Portable tanks having a capacity of 100 gallons or more
  2. Dry bulk tank vehicles
  3. Transit mix trucks and cement mixers

2: With tank vehicles 'Outage' means?

  1. An electrical failure when the engine is shut off
  2. Sparks that fly up from the pavement when touched by a ground wire
  3. Space that must be left in a cargo tank for liquids to expand

3: Why should you find out the 'Outage' for the liquids you are transporting?

  1. Different liquids expand by different amounts.
  2. You can not operate a tank if the electrical system is not working.
  3. A liquid that has frozen is no longer considered to be a liquid.

4: What are two problems especially important to tanker operation?

  1. A high center of gravity and the liquid surge of the cargo transported
  2. A low center of gravity and leakage from tanks
  3. The tendency of milk to spoil and driver inability to see into the tank

5: What is a tank compartment?

  1. A tank for storing hoses and couplings
  2. A liquid-tight division of a tank
  3. A wall with many openings

6: How is a bulkhead different from a baffle?

  1. Bulkheads are larger than baffles.
  2. Bulkheads are liquid-tight and baffles have holes in them
  3. The brakes will work better

7: Which is a factor that determines the amount of liquid you can load in a tank?

  1. The failure of molasses to flow properly in cold weather
  2. The height of the tractor
  3. The weight of the liquid

8: One of these helps determine the amount of liquid you can load in a tank

  1. The legal weight and load limits of your vehicle
  2. The wheelbase between your front and second axles
  3. The number of pumps that can be used to load your tank

9: Which of these helps determine the amount of liquid you can load in a tank?

  1. The size of the opening in the dome lid
  2. The amount that the liquid might expand in transit
  3. The maximum speed at which you will be driving

10: Why is the danger of liquid surge more when a tank is less than full?

  1. Liquid has more room to move in a partially-filled tank.
  2. The air pressure is greater inside a partially-filled tank.
  3. Many semi trailer tanks are slanted so that the lower end is at the back.

Question Your Answer Correct Answer

Cargo Test B

1: The side-to-side surge of liquid in a tank can cause a vehicle to?

  1. Stop instantly
  2. Bend at the fifth wheel connection
  3. Roll over

2: Do the speed limits posted for on-ramps and off-ramps apply to tankers?

  1. Yes, all limits are set with tankers in mind
  2. No, tankers are exempt.
  3. Not necessarily

3: Are smooth bore tankers different to drive than tankers with tank baffles?

  1. No, except that smooth bore tanks usually transport food products.
  2. Yes, forward and back surge is strong in smooth bore tankers.
  3. No, but tanks with bulkheads or baffles are harder to keep clean.

4: Baffles offer little resistance to ?

  1. Front-to-back surge
  2. Back-to-front surge
  3. Side-to-side surge

5: In what direction does liquid surge cause a vehicle to move?

  1. Forward and much faster
  2. Backward and much slower
  3. It will tend to move the vehicle in the direction that the liquid moves.

6: As compared to a smooth bore tank, a tank with bulkheads or baffles?

  1. Will have less front-to-back surge
  2. Will have less side-to-side surge
  3. There will be no difference

7: Why are unbaffled tanks used to transport food products?

  1. Sanitation regulations often forbid the use of baffles.
  2. Food products are lighter than gasoline and require less support.
  3. The baffles cause syrups to congeal.

8: The amount that a liquid can expand in transit can be affected by?

  1. The grade that is being pulled
  2. The suddenness with which the brakes are applied
  3. Air pressure is converted into hydraulic pressure

9: What is a reason for filling a tank only partially?

  1. The shipper ordered a small quantity.
  2. The shipment was too light in weight.
  3. The tank had a hole that leaked liquid.

10: Why is the stopping of a tank vehicle on a slippery surface extra hazardous?

  1. Liquid surge will tend to shove the vehicle ahead.
  2. A vehicle on a slippery surface can become stuck more easily.
  3. The wave of the liquid inside the tank may hit the rear of the tank.

Question Your Answer Correct Answer

Cargo Test C

1: What is overturn protection?

  1. Guards to protect fittings and valves in case of a rollover
  2. Weight reducers that will cause the trailer to ride lower
  3. Guards that keep a tractor from turning too sharply and breaking air lines

2: When you load a tank that has bulkheads, it is most important to check?

  1. The size of the hose connections
  2. The size of the bulkheads
  3. Weight distribution of the commodity being loaded

3: What is a dome cover?

  1. A cover to protect an opening where a tank is filled
  2. A cover to protect a drain on the bottom of the tank
  3. An extra cover to protect the cab in case of a rollover

4: If transporting flammable cargo over a railroad track you may shift gears?

  1. When the sound of your engine tells you it is time to shift gears
  2. When your tachometer shows 1100 RPM or higher
  3. Do not shift gears when crossing over railroad tracks.

5: How soon can you remove placards when you unload flammables from a tanker?

  1. After you have completed unloading, but before you drive away
  2. Before you start to unload or during the unloading
  3. After the tank has been cleaned or another commodity has been loaded

6: What can liquid surge do to the handling of a tank vehicle?

  1. Surge improves your ability to turn corners tighter.
  2. Surge may move the vehicle in the direction the liquid moves in the tank.
  3. Surge causes tank contamination.

7: A vehicle with empty tanks may?

  1. Be stopped more quickly with the emergency brakes
  2. Require a greater stopping distance than a loaded tanker
  3. Have better traction at all times than does a loaded tanker

8: Which is true about the emergency steering of tankers?

  1. Do not apply the brakes when making a quick turn.
  2. Countersteering is easy with a loaded tanker.
  3. Never try to stop a tanker without turning.

9: When loading a tanker you must consider weight distribution because?

  1. You do not want to put too much weight on the front or on the rear.
  2. Computers will not check this for you.
  3. Tanks can not be bent or strained.

10: A 'tank vehicle' includes any portable tank having a liquid capacity of?

  1. 100 gallons or more
  2. 500 gallons or more
  3. 1,000 gallons or more

Question Your Answer Correct Answer

Combination Test A

1: To prevent rollovers, remember to:

  1. Keep the load centered
  2. Keep the load as low as possible
  3. Go slow around turns
  4. All of the above

2: Large combination vehicles take longer to stop:

  1. When they are empty
  2. When they are loaded
  3. Than bobtail tractors
  4. It takes the same time whether empty or loaded

3: When traveling under 40 mph, you should follow other vehicles:

  1. Only during the day
  2. With a distance of one second for every ten feet of your vehicle length
  3. With a distance of one second for every twenty feet of your vehicle length
  4. As close as possible so others don't cut in

4: The earliest way to notice a trailer skid is:

  1. By feeling the pull on your steering wheel
  2. By feeling a jerking in the back of the tractor
  3. By seeing it in your mirrors
  4. By hearing the squeal from the tires

5: If you start a trailer skid you should:

  1. Step on the brakes quickly
  2. Release the brakes to get traction back
  3. Use the trailer hand brake
  4. Slowly apply pressure to the brakes

6: Offtracking" occurs when you go around a corner and:

  1. The rear wheels follow a different path than the front wheels
  2. The rear trailer goes off the road
  3. The rear wheels follow the front ones in a straight line
  4. The front of the trailer pulls to the right

7: The trailer hand valve should be used:

  1. Only at highway speeds
  2. Only when parking
  3. Only to test the trailer brakes
  4. Both B & C above

8: Which one of the following statements about tractor protection valves is NOT true?:

  1. It keeps air in the tractor in the event the trailer air leaks
  2. It is controlled by the trailer air supply valve in the cab
  3. When it closes it allows air to leak from the tractor line
  4. It will close automatically if air pressure is low

9: The trailer air supply control:

  1. Is a red, triangular-shaped knob
  2. Is a yellow, square knob
  3. Is a red, square knob
  4. Is a red, eight sided knob

10: Which one of the following is controlled by the foot brake?

  1. The service air line
  2. The emergency air line
  3. The shut off valve
  4. The hose coupler

Question Your Answer Correct Answer

Combination Test B

1: Before driving away, how can you always test the trailer brakes?

  1. With the hand valve or by pulling the tractor protection valve and then pulling against the brakes in a low gear.
  2. With the hand valve or by pulling the tractor protection valve, and accelerating.
  3. With the hand valve or by pulling the tractor protection valve, and pulling against the brakes gently at a maximum speed.

2: How are the air tanks on trailers and converter dollies filled?

  1. By the tractor protection valve.
  2. By the air compressor cut off valve.
  3. By the emergency supply line from the tractor.

3: What tells how much pressure the relay valves should send to the trailer brakes?

  1. The position of the foot pedal.
  2. The position of the brake linings.
  3. The pressure in the service line.

4: In what position should the air line shut off valves be when you check them?

  1. They should be in the open position except at the back of the last trailer.
  2. The should be in the open position.
  3. They should be in the closed position.

5: What should you use to be safe when you park trailers that don't have spring brakes?

  1. Use wheel chocks
  2. Use available rocks.
  3. Use the curb.

6: What happens if (only) the service line comes apart while you are driving?

  1. You may not notice anything until you try to put the brakes on when air loss from the leak will quickly lower the air tank pressure
  2. Nothing will happen.
  3. You will hear a loud explosion.

7: Why must the fifth wheel plate have enough grease?

  1. To prevent sparks - and smoke.
  2. To prevent squealing.
  3. To prevent steering problems.

8: Before backing up how should you position the tractor in relation to the trailer?

  1. Slightly to the left of the trailer.
  2. Slightly to the left of the center of the trailer.
  3. Directly in front of the trailer.

9: Before you back under a semi-trailer in coupling your tractor at what height should the trailer be?

  1. Four feet eight inches.
  2. Well above the top of the fifth wheel.
  3. Just below the middle of the fifth wheel.

10: You have connected the air lines but have not yet backed under the trailer. What should you do?

  1. Check the light connections.
  2. Supply air to the trailer electrical system.
  3. Supply air to the trailer system.

Question Your Answer Correct Answer

Combination Test C

1: Describe what the service line is for:

  1. To carry air to the parking brake to control the trailer brakes.
  2. To carry air to a relay valve that allows the trailer brakes to be applied quickly.
  3. To carry air to the cab of the rig to cool it.

2: What are other names for the Service Air Line?

  1. Control line or signal line
  2. Straight line or control line
  3. Curved line or signal line

3: What is the emergency air line?

  1. Air lines connecting the parking brake to the trailer brakes.
  2. Air lines that control the emergency brakes of the tractor.
  3. Air lines that control the emergency brakes on combination vehicles.

4: What is the emergency air line for?

  1. To engage the front brakes of the tractor in case of an emergency.
  2. To engage the trailer brakes in case of air pressure loss.
  3. To have extra air pressure in case you need it.

5: Why should you use chocks when parking a trailer without spring brakes?

  1. Because you don't want the trailer rolling down a hill.
  2. Because if the air pressure leaks away or down, there will be no brakes.
  3. Because some states require checking of trailer wheels.

6: The dolly tow bar may fly up if you unlock the pintle hook with the converter dolly still under the rear trailer.

  1. TRUE
  2. FALSE

7: You should use different methods for uncoupling second and third trailers.

  1. TRUE
  2. FALSE

8: Coupling and uncoupling methods are the same for more common tractor-trailer combinations, but there are other ways of coupling and uncoupling the many types of truck-trailer combinations that are in use.

  1. TRUE
  2. FALSE

9: There are more things to inspect on a combination vehicle than on a single vehicle.

  1. TRUE
  2. FALSE

10: Should you do a walk around inspection when preparing to pull Doubles or Triples

  1. Yes
  2. No

Question Your Answer Correct Answer

Combination Test D

1: Combination vehicles are usually heavier, longer and require more driving skill than single commercial vehicles.

  1. TRUE
  2. FALSE

2: More than half of truck driver deaths in crashes are the result of truck rollovers. Trucks turn over easier when fully loaded and are:

  1. Easy to get turned back right.
  2. Five times more likely to roll over in a crash than empty rigs.
  3. Ten times more likely to roll over in a crash than empty rigs.

3: The following two things will help prevent rollovers.

  1. Keep the cargo close to the front and drive slowly around turns.
  2. Keep the cargo as close to the ground as possible and drive slowly around turns.
  3. Keep the cargo to the back of the rig and drive the speed limit.

4: When you turn suddenly while pulling doubles, which trailer is most likely to turn over?

  1. The rear trailer is twice as likely to turn over as the tractor.
  2. The front trailer is twice as likely to turn over as the tractor.
  3. Each trailer is equally likely to turn over as is the tractor.

5: Why should you not use the trailer hand brake to straighten out a jackknifing trailer?

  1. Because the brake handle is too hard to reach.
  2. Because the brakes on the trailer wheels cause the skid in the first place.
  3. Because the brakes on the trailer wheels will not respond to the hand brake.

6: What is off tracking?

  1. When a vehicle goes around a corner, the rear wheels follow a different path than the front wheels
  2. When a vehicle goes around a corner, and the driver pulls off the road.
  3. When a vehicle goes around a curve and turns back the other way.

7: What is a "trailer jacknife?"

  1. When the tires on a trailer blow out.
  2. When the wheels of a trailer lock up and the trailer swings around.
  3. Reduce the risk of water in the air brake valves in warm weather
  4. When the wheels of a trailer lock up and the trailer pulls the tractor to a stop.

8: Should you swing wide to the left before starting a 90 degree turn?

  1. Yes
  2. No

9: Why should you not use the trailer hand valve while driving?

  1. Because it won't work as well as the foot brake.
  2. Because you should use the parking brake.
  3. Because of the danger of making the trailer skid.

10: Describe what the trailer air supply control does.

  1. It is used to supply the trailer with air, shut the air off and put on the trailer emergency brakes.
  2. It is a yellow, six sided knob used to control the tractor protection valve.
  3. It is a device used to keep the trailer behind the tractor.

11: Describe what the service line is for:

  1. To carry air to the parking brake to control the trailer brakes.
  2. To carry air to a relay valve that allows the trailer brakes to be applied quickly.
  3. To carry air to the cab of the rig to cool it.

12: What are other names for the Service Air Line?

  1. Control line or signal line
  2. Straight line or control line
  3. Curved line or signal line

13: What is the emergency air line?

  1. Air lines connecting the parking brake to the trailer brakes.
  2. Air lines that control the emergency brakes of the tractor.
  3. Air lines that control the emergency brakes on combination vehicles.

14: What is the emergency air line for?

  1. To engage the front brakes of the tractor in case of an emergency.
  2. To engage the trailer brakes in case of air pressure loss.
  3. To have extra air pressure in case you need it

15: Why should you use chocks when parking a trailer without spring brakes?

  1. Because you don't want the trailer rolling down a hill.
  2. Because if the air pressure leaks away or down, there will be no brakes.
  3. Because some states require checking of trailer wheels.

Question Your Answer Correct Answer

Combination Test E

1: Before you back under the trailer, you should be sure that the trailer brakes are?

  1. Disengaged (Unapplied).
  2. Locked (Applied).
  3. Unhooked.

2: When a tractor and a trailer are coupled, how much space should there be between the upper and lower fifth wheel.

  1. No space.
  2. Space enough to allow air to flow freely.
  3. Seven eight's inch.

3: After you lock the kingpin into the fifth wheel how can you check the connection?

  1. By tapping it with a rubber hammer.
  2. By pulling the tractor ahead gently with the trailer brakes locked.
  3. By pulling it by hand.

4: . Looking into the back of the fifth wheel, which part of the kingpin should the locking jaws close around?

  1. The skid plate.
  2. The pickup ramp.
  3. The shank of the kingpin.

5: For the coupling to be complete for a fifth wheel with a locking lever, where must the safety catch for the fifth wheel locking lever be?

  1. Over the locking lever.
  2. Level with the locking lever.
  3. Under the locking lever.

6: After you have coupled the tractor with a trailer where should the landing gear be before driving away?

  1. Raised at least six inches above the ground.
  2. Raised at least one foot above the ground with the crank handle secured in its bracket.
  3. Fully raised with the crank handle secured in its bracket.

7: Inspect the coupling. You must make certain that there is enough clearance between which two of the following?

  1. Between the fifth wheel and the floor of the trailer.
  2. Between the tractor taillights and the landing gear.
  3. Between the tops of the tractor tires and the nose of the trailer.

8: If you will be pulling doubles where one trailer is more heavily loaded than the other, where should the heavier loaded trailer be placed?

  1. It should always be behind the lighter trailer.
  2. It should always be in front of the lighter trailer
  3. It should be placed where it will have the least wind resistance.

9: What should the trailer height be before you connect a converter dolly to a second or third trailer?

  1. Five feet eight inches.
  2. It must be slightly lower than the center of the fifth wheel.
  3. Four feet eight inches.

10: Why should you never unlock the pintle hook with the dolly still under the rear trailer?

  1. The dolly tow bar may fly up.
  2. There may not be enough grease on the pintle hook to enable it to ride smoothly.
  3. It may be too hard to unlock it next time.

Question Your Answer Correct Answer

Combination Test F

1: What is the cause of more than half the truck driver deaths in crashes?

  1. Rush hour traffic.
  2. Truck rollovers.
  3. Heart attacks.

2: Are fully loaded rigs more likely to roll over in a crash than empty rigs?

  1. There is no difference as to the load so far as rolling over is concerned
  2. Fully loaded rigs are less likely to roll over in a crash.
  3. Fully loaded rigs are 10 times more likely to roll over in a crash.

3: What two things can a driver do to prevent rollovers?

  1. Keep the cargo as close to the ground as possible. Go slowly around turns.
  2. Go slowly around turns. Try to raise the center of gravity of your load.
  3. Accelerate slowly. Slow down quickly.

4: When you make a quick lane change, the trailer only may overturn. What is the effect called that produces this result?

  1. The crack-the-whip effect.
  2. The worm effect
  3. The hula effect.

5: Since a sudden movement with your steering wheel can tip a trailer over, how far behind other vehicles should you follow when pulling trailers.

  1. At least one second for each unit of your vehicle plus another second if going over 45 miles per hour.
  2. At least one second for each 20 feet of your vehicle length, plus another second if going over 35 miles per hour.
  3. At least one second for each ten feet of your vehicle length, plus another second if going over 40 miles per hour.

6: What are some things that you can do to avoid getting surprised and having to make a sudden lane change?

  1. Use your highbeam headlights at all times. Look far ahead all the time.
  2. Slow down to a safe speed before going into a turn. Focus your attention on the taillights of the vehicle ahead.
  3. Look far enough down the road and allow lots of following distance.

7: Why do large combination vehicles that are empty take longer to stop than when they are fully loaded?

  1. When lightly loaded, stiff suspension springs and strong brakes give poor traction and make it easy to lock up the wheels.
  2. Combination vehicles are longer than bobtails and therefore require more length to stop.
  3. Empty trucks are harder to control and the brake connections are more likely to come loose.

8: Why must you be extra careful when bobtailing?

  1. Bobtails may not be equipped with splash guards or back bumpers.
  2. Bobtails stop more smoothly but take longer to stop.
  3. Bobtails can be hard to stop smoothly and take longer to stop than a loaded tractor-semi trailer.

9: When the wheels of a trailer lock up, what will the trailer tend to do?

  1. The trailer will immediately stop.
  2. The trailer will tend to jackknife or swing around.
  3. The trailer will automatically break loose from the fifth wheel assembly.

10: Why should you check your mirrors whenever you apply the brakes hard?

  1. This is the best way to recognize if the trailer has started to skid.
  2. To tell if the mirrors are still in adjustment.
  3. To tell if your cargo door comes open.

Question Your Answer Correct Answer

Combination Test G

1: The service air line is controlled by:

  1. The pressure applied to the line by the foot brake or the trailer hand brake
  2. The relay valve which connects the trailer air tanks to the trailer air brakes
  3. The relay valve which sends air pressure from the trailer air tank to the trailer brake chambers
  4. All of the above

2: A device which connects the service and emergency air lines from the truck to the trailer is called:

  1. Relay switch
  2. Glad hands
  3. Service couplers
  4. Shut off valves

3: When coupling the glad hands, you should NOT do the following:

  1. Clean the couplers before a connection is made
  2. Press the two seals together at a 90 degree angle
  3. Cross air supply lines to test them for leaks
  4. Match the colors to each other, or read the tags before connecting

4: If you cross connect the two air lines the following will result:

  1. Nothing, the lines are interchangeable
  2. You will not have air going to the trailer brakes
  3. Your tractor brakes will lock up
  4. You could not move the tractor

5: The fifth wheel should be checked to see if:

  1. There is damage or missing parts
  2. It is properly greased
  3. The jaws are open
  4. All of the above

6: When coupling the trailer, back up slowly until:

  1. The load shifts inside the trailer
  2. The fifth wheel just touches the trailer
  3. You can see the rear wheels in the mirror
  4. You hit the front of the trailer with the tractor

7: In order to supply air to the trailer, you must first:

  1. Push in the tractor protection valve.
  2. Pull out the "air supply' knob
  3. Stop engine so you can hear the brakes
  4. Cross the air lines to check for leaks

8: A king pin is:

  1. The pin which locks the air brake lines together
  2. A pin which locks the trailer to the tractor on the fifth wheel
  3. A pin which holds the wheels on the axles
  4. The pin a driver receives for safe driving

9: In order to inspect the coupling you should:

  1. Make sure there is space between the upper and lower fifth wheel
  2. Check that the locking lever is in the open position
  3. Make sure the fifth wheel jaws are open
  4. Use a flashlight if necessary

10: You should crank the landing gear until:

  1. It is six inches off the ground
  2. You can clear railroad tracks
  3. The crank handle is loose
  4. It is as far up as it will go

Question Your Answer Correct Answer

Double-Triple Trailers

Sample Test A

1: If your trailer has started to skid, what should you do to get traction back?

  1. Release the brakes. Don't use the trailer handbrake.
  2. Press harder on the brake pedal to stop the vehicle faster
  3. Set the trailer handbrake

2: What is it called when the rear wheels follow a different path than the front wheels when a vehicle goes around a corner?

  1. A close fit.
  2. Offtracking
  3. Crack-the-whip effect

3: What are two reasons why doubles and triples take up more space than other commercial vehicles?

  1. They are longer and they cannot be turned or stopped suddenly
  2. They are longer and weave from side to side
  3. They are longer and are taller than tractors.

4: In allowing more space, what should you do before entering or crossing traffic?

  1. Move quickly because you have a deadline
  2. Because of the large size of your vehicle expect others to see you and make room for you
  3. Make sure there are large enough gaps before entering or crossing traffic

5: What is a reason why there is more chance of skids with doubles and triples in bad weather, slippery conditions and mountain driving?

  1. Your drive wheels provide security for the pulling unit only
  2. Your front wheels may not be equipped with brakes
  3. You have more length and more dead axles to pull

6: When can you use the trailer hand valve for parking a combination vehicle?

  1. Always.
  2. When the emergency brake is not working
  3. Never.

7: In normal driving should you ever use the trailer hand valve before the brake pedal to prevent trailer skids?

  1. Always
  2. When the brake pedal does not work.
  3. Never.

8: Which valve keeps air in the tractor or truck if the trailer breaks away or develops a bad leak?

  1. The one-way check valve
  2. The relay valve
  3. The tractor protection valve

9: If the tractor protection valve closes, letting air out of the trailer emergency line, which brakes should come on?

  1. The trailer handbrake
  2. The trailer emergency brakes
  3. The brakes on the front axle

10: What happens when you push in the trailer air supply control knob and what happens when you pull it out?

  1. Push it in to supply the trailer with air. Pull it out to release all air to the trailer
  2. Pull it out to release all air to the trailer. Do not push it in when the vehicle is underway
  3. Push it in to supply the trailer with air. Pull it out to shut the air off and put on the trailer emergency brakes

Question Your Answer Correct Answer

Sample Test B

1: On older vehicles there may be a lever, rather than a knob, for the tractor protection valve. What are the normal and emergency positions?

  1. The normal position is used for pulling a trailer. The emergency position is used to shut air off to the trailer
  2. The normal position is horizontal. The emergency position is vertical
  3. The normal position is used for supplying air to the drive wheel brakes. The emergency position is used to substitute for the emergency brakes

2: Which valves on the service line on the trailer connect the trailer air tanks to the trailer air brakes

  1. One-way check valves
  2. Relay valves
  3. Automatic valves

3: What is one purpose of the emergency air line?

  1. It allows you to continue your trip when the primary system is malfunctioning
  2. It activates an emergency locator transmitter
  3. It supplies air to the trailer air tanks

4: What happens to the trailer emergency brakes if there is a loss of pressure in the emergency line?

  1. The emergency trailer brakes will fail
  2. The emergency trailer brakes will fade
  3. The emergency trailer brakes will come on

5: When the emergency line loses pressure, what happens to the tractor protection value?

  1. The valve will open
  2. The valve will close
  3. The valve will pop out

6: If the air lines on a truck are color coded, which lines are blue?

  1. The emergency lines
  2. The spring line.
  3. The service line

7: To connect the glad hands, the tow seals are pressed together with the couplers at what kind of angle to each other?

  1. A 45 degree angle
  2. A 90 degree angle
  3. A 180 degree angle

8: Why should you lock the glad hands to each other when you are not towing any trailers?

  1. To prevent brake fade.
  2. To keep the brakes from overheating
  3. To keep dirt and water out the lines

9: If the spring brakes do not release when you push the trailer air supply control, what should you do?

  1. Unhook the air line connections
  2. Check the air line connections
  3. Tighten the air line connections

10: Do trailers made before 1975 have spring brakes?

  1. Most of them do.
  2. All of them do.
  3. Many do not

Question Your Answer Correct Answer

Sample Test C

1: Is it important to be careful when driving doubles and triples?

  1. TRUE
  2. FALSE

2: Special care should be taken when pulling two or three trailers because more things can go wrong and are less stable than other commercial vehicles

  1. TRUE
  2. FALSE

3: Doubles and triples are not as likely to turn over as are other combination vehicles

  1. TRUE
  2. FALSE

4: Is it alright to not check all the critical parts when doing a pretrip inspection on doubles or triples?

  1. Yes
  2. No

5: When driving doubles or triples how whould they be driven to avoid rollover or jackknifing?

  1. Fast
  2. Smoothly
  3. Slowly

6: Doubles and triples take up more space and are longer than other commercial vehicles, but do they need more space to turn?

  1. Yes
  2. No

7: For the safest handling on the road when pulling doubles and triples how should the load be positioned?

  1. The lighter load in first position, the heavier in the rear
  2. The heavier load in first position, the lighter in the rear.
  3. It does not matter which trailer is heavier
  4. You should equalize the load

8: What is a converter dolly used for?

  1. To couple tow or three trailers behind a tractor truck.
  2. To help move cargo in a semi-trailer
  3. To couple a trailer to a tractor.
  4. None of the above

9: Do all converter dollies have spring brakes?

  1. Yes
  2. No

10: What three methods can you use to secure a second trailer before coupling?

  1. Release dolly brake by opening the air tank petcock or if the dolly has spring brake, use the dolly parking brake control.
  2. Wheel the dolly into position by hand so it is in line with the kingpin.
  3. Use the tractor and first semi trailer to pick up the converter dolly and move it into position
  4. All of the above.

Question Your Answer Correct Answer

Sample Test D

1: The dolly tow bar may fly up if you unlock the pintle hook with the converter dolly still under the rear trailer

  1. TRUE
  2. FALSE

2: You should use different methods for uncoupling second and third trailers.

  1. TRUE
  2. FALSE

3: Coupling and uncoupling methods are the same for more common tractor-trailer combinations, but there are other ways of coupling and uncoupling the many types of truck-trailer combinations that are in use

  1. TRUE
  2. FALSE

4: There are more things to inspect on a combination vehicle than on a single vehicle

  1. TRUE
  2. FALSE

5: Should you do a walk around inspection when preparing to pull Doubles or Triples?

  1. Yes
  2. No

6: To test the trailer service brakes you should:

  1. Lightly test brakes at 5 mph
  2. Apply hand control and check to see if you feel the trailer brakes come on
  3. Depress the brake pedal

7: When pulling triple trailers, which unit is more likely to roll over:

  1. The front trailer
  2. The middle trailer
  3. The last trailer
  4. The tractor

8: The stopping distance for a tractor trailer traveling 55 mph on dry pavement is:

  1. Longer than a football field, about 300 feet
  2. Two football fields, about 600 feet
  3. Two thirds of a football field, about 200 feet
  4. Half of a football field, about 150 feet

9: A drive-wheel skid when pulling doubles could result in a jackknife when:

  1. The tractor brakes lock up
  2. The trailer pushes the towed vehicle sideways
  3. The trailer slides off the right side of the road
  4. The tractor spins around

10: When steering to avoid a crash while pulling doubles, remember to:

  1. Keep both hands on the steering wheel
  2. Stay off the brake while turning
  3. Be prepared to counter-steer
  4. All of the above

Question Your Answer Correct Answer

Sample Test E

1: To uncouple the converter dolly you should:

  1. Disconnect safety chains, lower the landing gear and chock the wheels
  2. Chock the wheels, lower the landing gear and disconnect safety chains
  3. Lower the landing gear, disconnect safety chains, and chock the wheels
  4. Chock the wheels, disconnect safety chains and lower the landing gears

2: It is dangerous to unlock the pintle hook with the dolly still under the rear trailer because:

  1. The dolly tow bar may fly up
  2. The brakes might release
  3. The kingpin might break off
  4. The fifth wheel might not be aligned

3: When inspecting doubles, the shut off valves at the rear of the trailer should be:

  1. Open at the rear of front trailer, open at the rear of the last trailer
  2. Open at the rear of front trailer, closed at the rear of the last trailer
  3. Closed at the rear of front trailer, closed at the rear of the last trailer
  4. None of the Above

4: To check for air flow to the last trailer:

  1. Pull forward and step on brake pedal
  2. Open emergency shut off valve and listen for escaping air
  3. Step on brake pedal and watch for brake lights
  4. Test emergency brake lines on level surface

5: You should test your tractor protection valve to avoid:

  1. The emergency brakes coming on due to loss of air
  2. The loss of air from the air hose
  3. The loss of air from your trailer brakes
  4. All of the above

6: To test the trailer service brakes you should:

  1. Lightly test brakes at 5 mph
  2. Apply hand control and check to see if you feel the trailer brakes come on
  3. Depress the brake pedal
  4. Release air to the brakes

7: When pulling triple trailers, which unit is more likely to roll over:

  1. The front trailer
  2. The middle trailer
  3. The last trailer
  4. The tractor

8: The stopping distance for a tractor trailer traveling 55 mph on dry pavement is:

  1. Longer than a football field, about 300 feet
  2. Two football fields, about 600 feet
  3. Two thirds of a football field, about 200 feet
  4. Half of a football field, about 150 feet

9: A drive-wheel skid when pulling doubles could result in a jackknife when:

  1. The tractor brakes lock up
  2. The trailer pushes the towed vehicle sideways
  3. The trailer slides off the right side of the road
  4. The tractor spins around

10: When steering to avoid a crash while pulling doubles, remember to:

  1. Keep both hands on the steering wheel
  2. Stay off the brake while turning
  3. Be prepared to counter-steer
  4. All of the above

Question Your Answer Correct Answer

Sample Test F

1: When pulling triple trailers, always put the heaviest trailer:

  1. In the front
  2. In the rear
  3. In the middle
  4. It doesn't make a difference

2: When coupling twin trailers, you should release the dolly brakes by:

  1. Dropping the trailer onto the fifth wheel
  2. Opening the air tank petcock
  3. Letting the air out of the brake line
  4. Pushing the air release valve on the dash

3: When connecting the converter dolly to the rear trailer, you should remember to:

  1. Make sure trailer brakes are locked
  2. Make sure trailer height is correct
  3. Raise the landing gear slightly off the ground
  4. All of the above

4: When coupling the rear trailer, the correct trailer height is:

  1. Six inches of space between the upper and lower fifth wheel
  2. Slightly lower than the fifth wheel
  3. When the locking jaws are closed around the kingpin
  4. The kingpin touches the fifth wheel

5: To uncouple double trailers, you should park your rig:

  1. On a slight incline
  2. On level ground
  3. In the company parking lot
  4. On a soft surface

6: To uncouple the converter dolly you should:

  1. Disconnect safety chains, lower the landing gear and chock the wheels
  2. Chock the wheels, lower the landing gear and disconnect safety chains
  3. Lower the landing gear, disconnect safety chains, and chock the wheels
  4. Chock the wheels, disconnect safety chains and lower the landing gears

7: It is dangerous to unlock the pintle hook with the dolly still under the rear trailer because:

  1. The dolly tow bar may fly up
  2. The brakes might release
  3. The kingpin might break off
  4. The fifth wheel might not be aligned

8: When inspecting doubles, the shut off valves at the rear of the trailer should be:

  1. Open at the rear of front trailer, open at the rear of the last trailer
  2. Open at the rear of front trailer, closed at the rear of the last trailer
  3. Closed at the rear of front trailer, closed at the rear of the last trailer
  4. None of the Above

9: To check for air flow to the last trailer:

  1. Pull forward and step on brake pedal
  2. Open emergency shut off valve and listen for escaping air
  3. Step on brake pedal and watch for brake lights
  4. Test emergency brake lines on level surface

10: You should test your tractor protection valve to avoid:

  1. The emergency brakes coming on due to loss of air
  2. The loss of air from the air hose
  3. The loss of air from your trailer brakes
  4. All of the above

Question Your Answer Correct Answer

Sample Test G

1: Is it important to be careful when driving doubles and triples?

  1. TRUE
  2. FALSE

2: Special care should be taken when pulling two or three trailers because more things can go wrong and are less stable than other commercial vehicles.

  1. TRUE
  2. FALSE

3: Doubles and triples are not as likely to turn over as are other combination vehicles.

  1. TRUE
  2. FALSE

4: Is it alright to not check all the critical parts when doing a pretrip inspection on doubles or triples?

  1. Yes
  2. No

5: When driving doubles or triples how whould they be driven to avoid rollover or jackknifing?

  1. Fast
  2. Smoothly
  3. Slowly

6: Doubles and triples take up more space and are longer than other commercial vehicles, but do they need more space to turn?

  1. Yes
  2. No

7: For the safest handling on the road when pulling doubles and triples how should the load be positioned?

  1. The lighter load in first position, the heavier in the rear.
  2. The heavier load in first position, the lighter in the rear
  3. It does not matter which trailer is heavier
  4. You should equalize the load

8: What is a converter dolly used for?

  1. To couple tow or three trailers behind a tractor truck
  2. To help move cargo in a semi-trailer
  3. To couple a trailer to a tractor
  4. None of the above

9: Do all converter dollies have spring brakes?

  1. Yes
  2. No

10: What three methods can you use to secure a second trailer before coupling?

  1. Release dolly brake by opening the air tank petcock or if the dolly has spring brake, use the dolly parking brake control
  2. Wheel the dolly into position by hand so it is in line with the kingpin
  3. Use the tractor and first semi trailer to pick up the converter dolly and move it into position.
  4. All of the above

11: The dolly tow bar may fly up if you unlock the pintle hook with the converter dolly still under the rear trailer.

  1. TRUE
  2. FALSE

12: You should use different methods for uncoupling second and third trailers.

  1. TRUE
  2. FALSE

13: Coupling and uncoupling methods are the same for more common tractor-trailer combinations, but there are other ways of coupling and uncoupling the many types of truck-trailer combinations that are in use.

  1. TRUE
  2. FALSE

14: There are more things to inspect on a combination vehicle than on a single vehicle.

  1. TRUE
  2. FALSE

15: Should you do a walk around inspection when preparing to pull Doubles or Triples?

  1. Yes
  2. No

Question Your Answer Correct Answer

General Knowledge

Sample Test A

1: How can you know you have the engine speed and road speed to shift gears?

  1. When the engine is lugging.
  2. By shifting whenever you notice heavy smoke coming from the exhaust.
  3. By using the sound of the engine to know when to shift.

2: What is true about downshifting before you reach a long downhill grade?

  1. It helps prevent the brakes from overheating and losing braking power.
  2. It puts an extra burden on the brake system.
  3. Starting downhill in low gear increases the chance of picking up speed.

3: When should you downshift for a curve?

  1. Before entering the curve.
  2. Upon entering the curve.
  3. After entering the curve.

4: What is the purpose of brake retarders?

  1. To help slow down the vehicle and to reduce brake wear
  2. To provide more traction on a slippery surface.
  3. To reduce brake wear and to reduce noise.

5: Should you turn the retarder off when the road is wet or snow covered?

  1. No because you need more braking power then
  2. No because the engine retarder will not affect traction.
  3. Yes the retarder may cause a skid.

6: How far ahead should you look while driving?

  1. Two seconds.
  2. Four seconds.
  3. 12 to 15 seconds.

7: What is a good reason for knowing what the traffic is doing on all sides?

  1. You need to have room to change lanes or stop.
  2. It is always necessary to know when you can make a U-turn.
  3. You need to eliminate all blind spots around you.

8: Should you always be looking into the distance ahead?

  1. Yes you should be prepared for all problems ahead.
  2. No you should shift your attention back and forth near and far.
  3. Yes by concentrating on the vehicle directly ahead you will be prepared.

9: What is a problem that you can have when using your mirrors?

  1. They never remain in the positions that you have placed them.
  2. They are of no help when you are changing lanes
  3. There are blind spots that your mirrors cannot show you.

10: Where do you place the three reflector triangles on a two-lane road?

  1. Within 10 feet of the rear and 100 feet to the rear and 100 feet from the front.
  2. Within 100 feet of the front and 500 feet from the front of the vehicle.
  3. Within 10 feet of the front and 100 feet to the front.

Question Your Answer Correct Answer

Sample Test B

1: Where do you place the three reflector triangles on a divided highway?

  1. Place them to the rear: within 10 feet and 100 feet and 200 feet.
  2. Place all of them in front of the vehicle up to five hundred feet
  3. Place two in front at ten feet and 100 feet and place one to the rear

2: What are three factors of total stopping distance with hydraulic brakes?

  1. Brake lag distance and pedal engaging distance and rolling distance.
  2. Reaction distance and application distance and braking distance.
  3. Perception distance and reaction distance and braking distance.

3: What should you do if your vehicle hydroplanes?

  1. Let up on the clutch.
  2. Release the accelerator and push in the clutch.
  3. Push down on the accelerator releasing the clutch.

4: How long does it take to stop a heavy vehicle going 55 mph on dry level road?

  1. About 100 feet . . . about six seconds.
  2. About 100 yards . . . about four seconds.
  3. About the length of a football field . . . about six seconds.

5: What is probably your best driving speed? Traffic is moving at 35 mph in a 55 mph zone.

  1. 55 miles per hour.
  2. 35 miles per hour.
  3. 45 miles per hour.

6: How does tire pressure affect hydroplaning?

  1. Hydroplaning is more likely to occur when tire pressure is low.
  2. Hydroplaning is not affected by tire pressure.
  3. Hydroplaning is more likely to occur when tires are over inflated.

7: How many times more distance does it take to stop when speed is doubled?

  1. Twice as much distance.
  2. Three times as much distance.
  3. Four times as much distance.

8: What is a good rule as to the speed you should go when driving at night

  1. Be able to stop within the distance that you can see with your low beams.
  2. Be able to stop within the range of your headlights.
  3. You should never drive so fast as to require your high beams.

9: When driving over 40 mph how much space should you try to keep in front of you?

  1. One second for each 15 feet of your vehicle length at speeds below 40 mph.
  2. At least one second for 10 feet of your vehicle length plus one second.
  3. With a forty foot vehicle leave three seconds

10: What are some things to do if you are being tailgated?

  1. Avoid quick changes of speed or direction.
  2. Try to reduce your following distance
  3. Speed up and flash your taillights on and off.

Question Your Answer Correct Answer

Sample Test C

1: Why can you never assume you will clear the heights posted at overpasses?

  1. Snow may have increased the clearances since the heights were posted.
  2. The weight of cargo can change height with an empty van being lower.
  3. Some roads can cause your vehicle to tilt.

2: You wish to turn right and you must swing wide. You?

  1. Start turning wide before you enter the turn.
  2. You may allow your rear wheels to climb over the curb.
  3. Turn wide as you complete the turn.

3: If you are turning left, which lane should you use of two left turn lanes

  1. Use either lane.
  2. Use the right hand lane.
  3. Use the left hand lane.

4: At night where can you look to avoid the glare of oncoming traffic?

  1. Close your eyes momentarily.
  2. Look at the centerline of the highway and watch for the dotted line.
  3. Try to look at the right side of the road and watch the sidelines.

5: What are some items to check especially before driving in winter weather?

  1. CB radio antenna.
  2. Coolant level and windshield washer and antifreeze.
  3. AM-FM radio.

6: How often should you check your tires when driving in very hot weather?

  1. Every 2 hours or every 100 miles.
  2. Every time you stop.
  3. Once each hour.

7: How do you lessen chances of having to make sudden moves to avoid hazards?

  1. Keep your vehicle centered in your lane by watching the white line.
  2. Watch far enough ahead so that hazards can be anticipated.
  3. Follow the driver ahead closely and watch his brake lights

8: What is a good policy for using your brakes when pulling off the road?

  1. Try to avoid using your brakes until your speed has dropped to about 20 m.p.h
  2. Brake hard immediately.
  3. Speed up to maneuver around.

9: In making a quick turn what is a point to remember?

  1. Do not apply the brake when you are turning.
  2. The brakes will prevent skidding in turns.
  3. Do not expect to counter-steer.

10: What is controlled braking?

  1. Brake so that your wheels will stop rolling and stop the vehicle quickly
  2. Apply your brakes fully and do not release them
  3. Applying the brakes as hard as you can without locking the wheels.

Question Your Answer Correct Answer

Sample Test D

1: What is the purpose of cargo blocking and bracing?

  1. To keep cargo from sliding and falling and getting out of balance.
  2. To keep containers from getting dirty.
  3. To support the trailer structure.

2: Must you show your logbook to officers?

  1. Yes
  2. No

3: How do you test hydraulic brakes for a leak with the vehicle stopped?

  1. Pump brake pedal three times - Apply firm pressure hold for five seconds
  2. Push down the brake pedal and do not release for at least five seconds
  3. Apply firm pressure - stopping the vehicle as quickly as possible.

4: How do you test hydraulic brakes for their stopping action?

  1. Push the brake pedal firmly - then hold for five seconds.
  2. Pump the brake pedal three times. Apply firm pressure
  3. Go about five miles per hour. Then push the brake pedal firmly

5: What is the minimum amount of tread depth that your tires should have?

  1. Four-thirty seconds inch tread depth in every major groove on all wheels
  2. One inch tread depth in every major groove on all wheels.
  3. Four-thirty seconds inch on front wheels -and two - thirty seconds on others.

6: When checking tires what are some problems that you should look for?

  1. Too much or too little specification information on the sidewalls.
  2. Bad wear - cuts or other damage - tread separation - cracked valve stems
  3. Regrooved or recapped or retreaded tires on the drive wheels

7: What are some steering system defects to look for?

  1. Missing nuts or bolts or cotter keys or other parts
  2. Steering wheel play of two degrees
  3. Steering wheel play of five degrees.

8: What are some defects to look for in the suspension system?

  1. Spring hangers that are cracked or broken.
  2. Oil leaks in the frame or fifth wheel assembly.
  3. Oil leaks in the brake drums.

9: In holding a steering wheel what is the proper way to place your hands?

  1. Loosely with at least one hand on the wheel.
  2. One hand at the top of the wheel and one hand at the bottom of the wheel.
  3. Firmly with your hands on opposite sides of the wheel.

10: What are some things to do when you are backing your vehicle?

  1. Look at your path. Back slowly. Back straight back.
  2. Insist on having a helper to guide you.
  3. Back and turn from the right whenever possible.

Question Your Answer Correct Answer

Sample Test E

1: Which of the following are causes of vehicle fires:

  1. Spilled fuel
  2. Short circuits
  3. Driver smoking
  4. All of the above

2: What is the first thing to do if your vehicle catches fire while driving?

  1. Open the door and jump out of the vehicle
  2. Pull off the road and park in an open area
  3. Find a service station to pull in to
  4. Open the hood and let the flames die down

3: Water can be used to extinguish which of the following fires?

  1. Tires
  2. Electrical fire
  3. Gasoline fire
  4. Chemical fire

4: The best way to put out a fire is to:

  1. Get as close as possible and spray the flames
  2. Stand upwind and spray the base of the fire
  3. Stand upwind and spray the base of the fire
  4. Stop spraying as soon as the flames disappear

5: What is the best thing to do if you are tired?

  1. Drink plenty of coffee to stay awake
  2. Take pills to keep you alert
  3. Plan your trips for the middle of the night to avoid traffic
  4. Get enough sleep

6: Which of the following statements about alcohol is true:

  1. If you eat before you drink you won't get as drunk
  2. Drinking coffee will help you stay sober
  3. Fresh air will help you sober up
  4. A few beers are the same as a few shots of whiskey or glasses of wine

7: If you do not have a HAZMAT endorsements on your CDL, under what conditions may you legally haul hazardous materials?

  1. If your dispatcher feels it is an emergency situation
  2. If the load does not require placards
  3. When you will remain within your state
  4. When a DOT official approves the load

8: If you are asked to haul a placarded load and you do not have HAZMAT endorsements you should:

  1. Obtain written approval from your dispatcher
  2. Obtain written approval from the DOT official
  3. Refuse to take the load
  4. Take the placards off the vehicle

9: You should check your cargo how often?

  1. Within 50 miles after beginning a trip
  2. After you have driven for three hours
  3. After you take a break
  4. All of the above

10: Overloading your trailer can result in the following EXCEPT:

  1. It can slow you down on upgrades
  2. It can increase stopping distances
  3. It can help you go through snow better
  4. It can increase speed on downgrades

Question Your Answer Correct Answer

Sample Test F

1: What are two factors in knowing when to shift?

  1. Using transmission speed and clutch stroke
  2. Using engine speed and road speed
  3. Using road speed and feel of the road

2: A pretrip inspection should be performed

  1. Each morning.
  2. Before each trip.
  3. Each evening at the end of the run.

3: What is a vehicle condition report?

  1. A list of problems found by you.
  2. A list of items that you want checked by a mechanic.
  3. A report of the condition of the fleet

4: What may rust around the wheel nuts mean?

  1. The wheel may be falling off or coming loose.
  2. There is water inside the tires.
  3. The wheel nuts may be loose.

5: Wheels or rims that have been repaired by welding are?

  1. Not safe.
  2. Ten times stronger than new wheels or rims.
  3. Are dangerous only when operated off highway in rough terrain.

6: Exhaust system parts should not?

  1. Be removed from a vehicle unless there has been damage from an accident.
  2. Be taken off a vehicle until the engine is overhauled.
  3. Rub against fuel system parts, tires or other moving parts of the vehicle.

7: If the cargo contains any hazardous materials you must check to see if?

  1. Placards are required.
  2. A bill of lading is required.
  3. A freight bill will have to be issued.

8: To keep your vehicle from rolling back when you start up you.

  1. Start off in a higher gear.
  2. Partly engage the clutch before you take your right foot off the brake.
  3. Engage the clutch fully and continue to hold it while starting off in a higher gear.

9: As soon as your see your trailer getting off the proper backing path you?

  1. Hold the steering wheel tighter.
  2. Turn the top of the steering wheel opposite the direction of the drift.
  3. Turn the top of the steering wheel in the direction of the drift.

10: When traction is poor as in rain or snow you speed up?

  1. Very gradually.
  2. Gradually to normal speed.
  3. Normal speed.

Question Your Answer Correct Answer

Sample Test G

1: When backing a trailer you turn the steering wheel?

  1. The same as you would for a car, straight truck or bus.
  2. Toward the direction you want to go.
  3. Opposite the direction you want to go

2: When backing a trailer you may make corrections to reposition your vehicle by?

  1. Pull-ups.
  2. Shutting off the engine.
  3. Pulling out and starting over.

3: Whether backing a straight truck or a combination vehicle you back and turn?

  1. Toward the right hand side.
  2. Toward the driver's side.
  3. Toward the passenger side if the passenger is a helper.

4: If you are using a helper when you are backing the helper should?

  1. Agree with you as to a signal for 'Stop' and stand in front of you.
  2. Agree with you as to a signal for 'Stop' and stand where you can hear him signal.
  3. Agree with you as to a signal for 'Stop' and stand where he can be seen by you.

5: Some newer vehicles have 'progressive' shifting which means?

  1. The vehicle has an automatic transmission.
  2. All of the speeds can be handled by shifting only once to move the vehicle.
  3. The RPM at which you shift becomes higher as you move up in the gears.

6: Downshifting requires knowing when to shift. To do this correctly you should?

  1. Use either the tachometer or speedometer.
  2. Use the brownie transmission.
  3. Have the habit of downshifting when you think you are at the right road speed.

7: Before starting down a hill you want to be sure that you are in the proper gear. You?

  1. Downshift just after you have started down the hill.
  2. Downshift after you have started down the hill and 'have got the feel' of the grade.
  3. Downshift before you start down the hill.

8: When going downhill in a vehicle with an automatic transmission you?

  1. Select a low range to get greater engine braking.
  2. Concentrate on other safety procedures since the shifting is being handled for you.
  3. Select a gear range that will take all stress off your engine.

9: When turned 'on' brake retarders apply their power whenever

  1. You let off the air brake system to the trailer.
  2. Whenever you let up all the way on the accelerator pedal.
  3. Whenever you push down on the accelerator pedal and let off drive wheel brakes.

10: Besides looking for vehicles coming into your lane, looking for traffic means?

  1. Watching for brake lights of slowing vehicles ahead.
  2. Shifting gears with your eyes fixed on the traffic straight ahead
  3. Always keeping your eyes fixed on the highway ahead.

Question Your Answer Correct Answer

Sample Test H

1: Besides watching the traffic behind, you can use your rear view mirrors to

  1. Watch for highway patrol vehicles that are following you.
  2. Watch your blind spots.
  3. Watch for possible tire fires.

2: In turning you should look in your mirrors to make sure that?

  1. Your tractor is entering the proper lane.
  2. Your fifth wheel connections adjusted properly.
  3. The rear of your trailer will not hit anything.

3: When merging with other traffic you should use your mirrors to make sure

  1. Your fifth wheel connection is operating properly
  2. Your are coming out of the proper lane to merge with this traffic.
  3. The gap in the traffic is large enough for you to enter.

4: To be sure that you know what is happening on the highway ahead of you?

  1. Don't focus on the mirrors for too long.
  2. Don't use your mirrors whenever your vehicle is moving forward.
  3. Have extra mirrors that show what is ahead of you.

5: Convex mirrors show a wider area than flat mirrors. But they also?

  1. Make everything seem larger
  2. Make everything seem farther away than it really is
  3. Maker everything seem closer than it is.

6: Two of the three good rules for using turn signals are?

  1. Signal early and signal continuously.
  2. Signal 100 feet before you start your turn and cancel the signal as you turn
  3. Signal 100 feet before you start your turn and cancel the signal as you turn

7: A third good rule for using turn signals is?

  1. Assume that the self-canceling signal will cancel your turn signal at the right time.
  2. Always cancel your signal before the self-canceling signal does
  3. If you don't have self-canceling signals don't forget to turn off the turn signals

8: When your need to slow down you may want to warn drivers behind you by?

  1. Tapping lightly on the brake pedal to flash the brake lights
  2. By turning on your clearance lights and tapping lightly on the brake pedal.
  3. By turning on your clearance lights and tapping lightly on the brake pedal.

9: Because of the size of your vehicle you may wish to flash your brake lights to?

  1. Remind drivers behind you that your vehicle is big.
  2. Remind drivers behind you that you are thinking about turning
  3. Remind drivers behind you of hazards that you see ahead

10: To make a tight turn with a large vehicle you may have to?

  1. Go slower than many car drivers would expect.
  2. Swing wide to the opposite side before you make the turn.
  3. Have the trailer wheels jump the curb.

Question Your Answer Correct Answer

Sample Test I

1: Should you have to stop your vehicle in the road to load cargo or passengers you?

  1. Should stop suddenly so as to get the attention of the drivers behind you
  2. Should slowly swerve from side to side to warn drivers behind you.
  3. Flash your brake lights to warn drivers behind you

2: When should you help out other drivers by signaling when it is safe for them to pass

  1. Always
  2. Whenever they signal you asking for your help.
  3. Never.

3: Whenever you are about to pass a vehicle, pedestrian or bicyclist you should?

  1. Assume that they have heard the sound of your vehicle even if they have not seen it.
  2. Assume that they have not seen you
  3. Assume that they will always do what they are supposed to do.

4: At dawn or dusk or in rain or snow, when it's hard for others to see you, you might?

  1. Turn on your clearance lights or parking lights.
  2. Turn on your high beam headlights.
  3. Turn on your low beam headlights.

5: . If you have to set out warning reflective triangles by the highway you hold them?

  1. Between yourself and the oncoming traffic
  2. Between yourself and your vehicle.
  3. On the upwind side of your vehicle

6: Slight melting will make ice wet. Which is more slippery?

  1. Wet ice.
  2. Ice that is not wet.
  3. Wet ice and ice that is dry are equally slippery.

7: Why do empty trucks usually require greater stopping distances?

  1. Empty trucks have less wind resistance.
  2. Empty trucks have more traction.
  3. Empty trucks can bounce and lock up their wheels.

8: Empty busses may not require more stopping distance than loaded busses because?

  1. They usually have as much braking power when empty as when loaded.
  2. Empty busses have more traction.
  3. They have no wind resistance.

9: What is 'black ice'?

  1. Thick layers of ice that contain mud and other debris
  2. Ice that remains only when the temperature has risen to above freezing
  3. A thin layer of ice so clear you can see the road underneath it

10: Which is a danger when traveling alongside other vehicles

  1. You may be trapped when you need to change lanes
  2. You may be unable to slow down when you need to
  3. You may not be able to accelerate when you need to.

Question Your Answer Correct Answer

Sample Test J

1: The amount of space you need to cross or to enter traffic is affected by?

  1. The weight of your load
  2. The size of your transmission
  3. The shape of your steering wheel

2: If a vehicle that is coming toward you has on high beams you should

  1. Turn on your high beams and leave them on until the vehicle has gone by.
  2. Flash your high beams on and off quickly and look off to the right edge of the road.
  3. Keep your lights on low beam and look off to the right.

3: You should use your high beam headlights?

  1. Whenever you can and the law allows it.
  2. Only when absolutely necessary.
  3. Until you are following within 1500 feet of another vehicle.

4: Before putting tire chains on your vehicle you check to be sure that?

  1. It is the time of year when the law specifies that chains be used.
  2. The chains have no broken hooks or cross links or bent or broken side chains.
  3. That you have enough chains to cover every tire on your vehicle.

5: Roads usually become more slippery when?

  1. The temperature is so high as to produce a warm rain.
  2. The temperature has risen to the point where the ice begins to melt.
  3. The temperature has dropped below the freezing point and the road is dry.

6: Antifreeze is effective?

  1. In cold weather only.
  2. In hot weather only.
  3. Under hot conditions as well as cold conditions.

7: With a pressurized cooling system you should not?

  1. Remove the radiator cap until the system has cooled.
  2. Rely on see-through coolant overflow containers.
  3. Remove the radiator cap unless you have a replacement available.

8: What should you do before you drive if you are feeling drowsy?

  1. Drink black coffee.
  2. Get some sleep.
  3. Walk around your vehicle several times.

9: Since air pressure increases with temperature increases you should?

  1. Let some air out of your tires if you have been driving for hours on a hot road.
  2. Check your tires to see if air pressure has increased and then let out some air.
  3. Leave the tires alone since the air pressure will decrease when the tires cool off.

10: Speed limits that are posted at freeway offramps may not be?

  1. High enough for trucks or busses.
  2. Safe speeds for larger vehicles or heavily loaded vehicles.
  3. High enough to allow you to turn and brake a heavy vehicle.

Question Your Answer Correct Answer

Sample Test K

1: If you are convicted of a traffic violation you should notify your employer within 30 days if the following is true:

  1. The violation occurred in your personal vehicle
  2. The violation occurred in a commercial vehicle
  3. The violation was for parking in a restricted area
  4. both a & b

2: IA commercial motor vehicle is defined as:

  1. A vehicle with a gross weight over 26,001 lbs.
  2. A vehicle transporting hazardous materials
  3. A vehicle transporting 16 or more passengers
  4. All of the above

3: Suspension systems are intended to:

  1. Keep the load in place
  2. Keep the axles in place
  3. Keep the brake drums from failing
  4. Keep the steering wheel tight

4: Why should the exhaust system be checked?

  1. A leaking exhaust system can allow poisonous fumes into the cab
  2. A leaking exhaust system can promote poor fuel mileage
  3. A leaking exhaust system can be caused by snow and rain
  4. A leaking exhaust system can hamper your visibility

5: Which of the following is NOT part of the pre-trip inspection:

  1. Check engine oil level
  2. Check horn(s)
  3. Check air seat adjustments
  4. Check seat belt fastener

6: Tires should be checked for wear often and should be replaced:

  1. When the tread separates
  2. When there are broken valve stems
  3. If the tread depth on your front tires is less than 4/32 deep
  4. All of the above

7: The proper way to hold your hands on the steering wheel is:

  1. Near the bottom of the wheel
  2. Opposite sides of the wheel
  3. Near the top of the wheel
  4. One hand on the steering wheel and one on the shifter

8: When backing up your tractor trailer you should try to AVOID:

  1. Backing toward the right (passenger) side
  2. Backing toward the left (driver's) side
  3. Pulling ahead to reposition your trailer
  4. Having someone help/guide you

9: When accelerating, which of the following is true?:

  1. Always use the parking brake if you need to slow down
  2. You should not engage the clutch before you take your foot off the brake
  3. You should speed up smoothly and gradually and avoid jerking
  4. Accelerate when your wheels spin in snow or rain

10: Which one of the following is true about shifting gears?:

  1. Remain in neutral as long as possible
  2. Allow the engine tachometer to rev into the highest range possible
  3. You should ignore engine sounds
  4. Understand the best RPM range for your vehicle

Question Your Answer Correct Answer

Sample Test L

1: Hydroplaning can occur when you drive through water or slush. If you experience hydroplaning you should:

  1. Lightly tap your brakes to slow down
  2. Release the accelerator and push in the clutch
  3. Steer hard to the right
  4. Speed up to get through the water quickly

2: Which of the following is NOT true when driving through a curve?

  1. You should slow down before the curve
  2. You should brake during the curve
  3. The higher your truck's center of gravity is, the easier it is to roll over
  4. You should be in a gear that allows you to accelerate during the curve

3: When driving in heavy traffic you should travel:

  1. At a speed consistent with the flow of traffic
  2. Five mph less than automobiles regardless of posted limits
  3. In the left lane when possible to avoid heavy lanes of traffic
  4. In the right lane to avoid passing

4: Safe drivers maintain space around their vehicles for the following reasons except:

  1. In case you must stop suddenly reasons except:
  2. In case there are cars following too closely
  3. In case there is an obstacle in the road
  4. In case you need to test your brakes

5: How much space should you maintain in front of you at highway speeds?

  1. Seven seconds for a 60 foot vehicle
  2. Five seconds for a 60 foot vehicle
  3. Ten seconds for a 60 foot vehicle
  4. Three seconds for a 60 foot vehicle

6: If you find that you are being tailgated, you should:

  1. Quickly change lanes to avoid an accident
  2. Flash your brake lights to warn the tailgater
  3. Decrease the distance between you and the tailgater
  4. Decrease your speed and Increase the distance between you and any vehicle ahead of you.

7: Which of the following statements about overhead space is true?:

  1. The heights posted at bridges are usually accurate
  2. The weight of the cargo changes a truck's height
  3. Warning lights are always installed on low bridges and overpasses
  4. An empty van is lower than a loaded one

8: When making a right hand turn you should always:

  1. Steer into the left lane first so you can make the corner
  2. Rush through the intersection so you can get out of the way of traffic
  3. Back up to make the driver behind you move back
  4. Keep the rear of your vehicle close to the curb

9: When making a left hand turn you should always:

  1. Wait until you reach the center of the intersection before you turn
  2. Start in the left hand lane if there are two turning lanes
  3. Pull into the intersection in case the light turns before you are through
  4. Wait for a small gap in traffic and accelerate

10: Driving at night is more dangerous because of the following EXCEPT:

  1. Most people are less alert at night due to fatigue
  2. Headlights often cause glare which can blind drivers
  3. Traffic lights are less visible than in the day
  4. There are more drunk drivers at night

Question Your Answer Correct Answer

Sample Test M

1: Which of the following statements about downshifting is NOT true?

  1. Downshift before starting down a hill
  2. Downshift before entering a curve
  3. Downshift when you get to the bottom of a hill
  4. Downshift before climbing a hill

2: An engine retarder is designed to:

  1. Slow the vehicle down without using brakes
  2. Apply braking power to the steering wheel
  3. Work only with manual transmissions
  4. Work when you accelerate

3: When driving at highway speeds, you should look 12-15 seconds ahead which is:

  1. At least 100 yards
  2. At least half a mile
  3. At least a quarter of a mile
  4. At least one tractor trailer length

4: Which of the following statements about your mirrors is true?:

  1. You cannot see your tires
  2. You should be able to see any cars overtaking you
  3. You should be able to see any vehicles behind your trailer
  4. You should be aware that some objects may appear smaller in your mirrors

5: When using turn signals, which of the following should be avoided?:

  1. Signal just before you start to make a turn
  2. Signal when you want to change lanes
  3. Signal before you exit
  4. Signal when you are merging into traffic

6: The best way to warn the drivers behind you that you are slowing down is to:

  1. Blow your horn
  2. Turn on your emergency flashers
  3. Tap your brakes
  4. Flash your bright lights

7: If you must stop on the shoulder of the road, you should place warning devices at all of the following places except:

  1. Within ten feet of the front or rear corners
  2. About 100 feet behind your vehicle
  3. About 100 feet in front of your vehicle
  4. On the other side of a divided highway

8: Which of the following statements about stopping distances is NOT true?

  1. Empty trucks require greater stopping distances because they have less traction
  2. Trucks traveling on ice or snow require greater stopping distances
  3. Brakes, tires, springs and shocks are designed to work best when the truck is empty
  4. Trucks traveling at higher speeds require greater stopping distances

9: At 55 miles-per-hour it takes about 6 seconds to stop a fully loaded truck and the braking distance is:

  1. About the length of a football field or 100 yards
  2. About 1/4 mile
  3. About two truck lengths
  4. About two car lengths

10: When is the road likely to be more slippery?:

  1. After a hard rain
  2. After it has been raining lightly for 30 minutes
  3. When it has just begun to rain
  4. After the rain has ended

Question Your Answer Correct Answer

Sample Test N

1: The following statements about an engine overheating are TRUE EXCEPT:

  1. Antifreeze is only used in colder temperatures
  2. You should never remove the radiator cap to allow heat to escape the radiator
  3. It is not advisable to drive without radiator fluid
  4. Never leave the engine running if it is overheating

2: An engine retarder is designed to:

  1. Use higher gears when you have a heavy load
  2. Allow the brakes to heat up for better stopping power
  3. Be in the right gear before starting down the hill
  4. Put heavy pressure on the brakes to allow the drums to cool

3: The best time to test your parking brake is:

  1. When the vehicle is parked
  2. When the vehicle is going down a hill
  3. When the vehicle is moving slowly
  4. When the vehicle is traveling on the highway

4: The following are all probable signs of a tire blowout EXCEPT:

  1. A loud bang
  2. Vibration of the vehicle
  3. The smell of smoke
  4. A thumping sound

5: In the event of a tire failure you should:

  1. Stay off the brake and hold the steering wheel firmly
  2. Pump the brakes until you come to a stop
  3. Accelerate quickly until you find a place to pull over
  4. Disengage the clutch so you are in neutral gear

6: Your vehicle is going down a long hill and your brakes begin to fail. What should you do?:

  1. Pump the brake pedal
  2. Downshift
  3. Put the gears in neutral
  4. Look for an escape ramp or an escape route

7: To correct a drive wheel acceleration skid you should:

  1. Stop accelerating and push in the clutch
  2. Downshift
  3. Pump the brake
  4. Accelerate quickly

8: To correct a drive-wheel braking skid you should:

  1. Pump the brake
  2. Steer in the opposite direction
  3. Release the brake and steer/counter-steer
  4. Accelerate quickly!

9: A front wheel skid is usually caused by:

  1. Driving too fast for conditions
  2. Lack of tread on the front tires
  3. Cargo improperly loaded
  4. All of the above

10: In the event of an accident you should remember to do the following EXCEPT:

  1. Move the victims out of the way immediately
  2. Protect the area
  3. Notify authorities
  4. Stop any bleeding and keep the victims warm

Question Your Answer Correct Answer

Hazardous Materials Endorsement

Sample Test A

1: Which must be shown on a hazardous materials bill of lading?

  1. The shipper's permit number
  2. The quantity of hazardous materials
  3. All of the carrier's state and federal permit numbers

2: What are placards?

  1. Diamond-shaped warning signs placed on vehicles
  2. Warning signs that are placed on boxes and other packages
  3. Stickers that must be displayed in the lower right side of windshields

3: Before transporting certain radioactive materials a route restriction?

  1. May be required if the shipper tells you about it
  2. Should be obtained
  3. May be necessary

4: Which is correct about listing hazardous materials on a bill of lading?

  1. The commodity is named in the hazardous materials column.
  2. All commodities must be named with large letters.
  3. The proper shipping name of the commodity is listed first.

5: What is the intent of the hazardous materials regulations?

  1. To communicate risk, to protect all, and to contain the product
  2. To reduce the costs of transportation.
  3. To raise funds for the government through fines for violations.

6: Two letters that may be shown before the name of a hazardous substance?

  1. AD
  2. AD
  3. RQ

7: Which is not part of a correct shipping name?

  1. NOS
  2. The technical name of the product
  3. The brand name of the product

8: Information on a hazardous materials shipping paper must be shown?

  1. Proper shipping name, then packing group, then ID number
  2. Proper shipping name, then hazard class, ID number, packing group
  3. Hazard class first followed by shipping name, ID and packing group

9: Which is not the proper four-digit Identification (ID) Number?

  1. USA number
  2. NA number
  3. UN number

10: If a label will not fit on a hazardous materials package?

  1. The driver may use a bright red marking pen to mark the package
  2. The package will not have to be labeled if it weighs less than 100 pounds.
  3. The label must be put on a tag which is then wired to the package

Question Your Answer Correct Answer

Sample Test B

1: Which is an acceptable method of showing shipper's certification on papers?

  1. Longhand
  2. Printed
  3. Red Ink

2: Drivers who transport highway route-controlled radioactive materials must have?

  1. Had special training within the past two years
  2. Special training from their local union every three years
  3. Special training before each trip

3: Which is an acceptable quantity shown on a hazardous materials bill of lading?

  1. 2,100 pounds
  2. 2100
  3. 2100

4: In bulk quantities only on highways marine pollutants are regulated?

  1. In interstate commerce
  2. In foreign commerce within all countries
  3. In intrastate and interstate commerce

5: You may load cylinders of compressed gas without racks if the cylinders?

  1. Can be loaded in rounded containers
  2. Can be loaded in an upright position without supports
  3. Can be loaded on the vehicle laying down flat and braced

6: A vehicle carrying hazardous materials must be parked at least?

  1. 300 feet away from an open fire
  2. 600 feet away from an open fire
  3. 900 feet away from an open fire

7: When placards are required, they should be placed?

  1. On the rear and both sides of the vehicle
  2. On the front, the rear, and both sides of the vehicle
  3. As long as four placards are displayed they may be placed as wanted

8: The Hazardous Materials Table is organized?

  1. In alphabetical order by the proper shipping name
  2. In numerical order by the hazard class number only
  3. In the same order as the listings in the placard tables

9: With tank vehicles the Identification (ID) Number must appear on?

  1. The tanks and the placards
  2. The shipping papers.
  3. The placards and the shipping papers

10: When a vehicle is parked the hazardous material shipping papers must be?

  1. Kept on the seat or in a pocket in the left door
  2. Kept under the seat
  3. In the glove compartment or in the driver's possession

Question Your Answer Correct Answer

Sample Test C

1: Labels on hazardous materials must measure?

  1. At least two inches on each side
  2. At least three inches on each side
  3. At least four inches on each side

2: If a shipper's certification is signed, must you accept a leaking package?

  1. You must accept the package, but report it to your dispatcher immediately
  2. You must refuse the shipment
  3. You must place the shipment inside a larger container that is not leaking

3: Which word is a hazardous class description found on labels?

  1. Corrosive
  2. Fragile
  3. Cancer

4: If required to stop for a railroad crossing you must stop?

  1. Twenty to forty feet before the nearest rail
  2. Fifteen to fifty feet before the nearest rail
  3. Wherever you can plainly see any approaching train

5: Corrosives could require more than one label because they could also be?

  1. Asbestos
  2. Cancer-inducing in rats
  3. Poison by inhalation

6: A commodity named in Placard Table I requires a placard for?

  1. Any quantity
  2. A shipment that weighs more than a thousand pounds
  3. A shipment that contains more than one class of hazardous materials

7: What is a marking that must be shown on liquid hazardous materials packages?

  1. This Side Up
  2. This Package May Leak
  3. This package Opens at the Bottom

8: For the placards to use with packaged freight you must know the hazard class?

  1. The amount shipped and the total weight of hazardous materials on board
  2. The amount shipped and the ID Number
  3. The amount shipped, the ID Number and the NMFC classification

9: Which of these is true?

  1. With hazardous materials there are no regulations against smoking
  2. You may smoke around Flammables if there is a fire extinguisher handy
  3. Do not smoke when around Explosives, Flammables and Oxidizers

10: When fueling a placarded vehicle, someone must be?

  1. Supervising the fueling from a safe distance
  2. Supervising the fueling from at least 25 feet away
  3. At the nozzle, controlling the fuel flow

Question Your Answer Correct Answer

Sample Test D

1: Which Code of Federal Regulations requires that placards be used?

  1. Title 13, CFR
  2. Title 27, CFR
  3. Title 49, CFR

2: When should a driver have all needed placards in place for packaged freight?

  1. Before loading
  2. After loading each commodity that requires a certain placard
  3. After the vehicle is loaded but before it goes out onto the highway

3: How should each trailer be placarded in a combination unit?

  1. At the front of the first trailer and the back of the last trailer
  2. For the class of hazardous material contained in that trailer
  3. On the sides of the trailers only with a placard on the power unit

4: When are you allowed to have overhang (tailgate) loads of Explosives?

  1. Never
  2. When there are two escort vehicles with one following
  3. During daylight hours and the overhang does not exceed five feet

5: An exception to the rule for trailer placarding for semis is?

  1. There is no exception to the rule that trailers must be placarded all around.
  2. AInstead of the trailer, a placard may be placed on the front of the tractor
  3. At least one placard must be placed on the front of any semi trailer.

6: Which requires a placard even if there is only one pound of it on board?

  1. Flammable Liquids
  2. Corrosives
  3. Class A Explosives

7: What placard may be used for Table 2 materials none of which equal 1,000 lbs?

  1. Dangerous
  2. Dangerous When Wet
  3. Hazardous

8: When driving a placarded vehicle you must stop and check the tires?

  1. Every 200 miles
  2. Every two hours or 100 miles
  3. Every hour or 55 miles

9: Cargo tanks or portable tanks loaded with hazardous materials must show ?

  1. The ID Number of the product
  2. A label on each side
  3. Warning: Flammable

10: The total transport index of all radioactive packages in a vehicle can't exceed?

  1. 25
  2. 50
  3. 100

Question Your Answer Correct Answer

Sample Test E

1: An X or an RQ shown in the HM column of a shipping paper means?

  1. The material in the shipment is a hazardous material.
  2. This is a hazardous material that any driver can transport
  3. This material is an exception to the hazardous material rules.

2: If special permits or special routes are required, who is responsible for this?

  1. Owner of the vehicle
  2. The company dispatcher
  3. Driver of the vehicle

3: The EPA registration numbers of the carrier, shipper and destination must be on?

  1. All bills of lading
  2. Material Safety Data Sheets
  3. Hazardous Waste Manifests

4: Transporters must keep a copy of hazardous waste manifests for at least?

  1. One year
  2. Two years
  3. Three years

5: Which may be exempt from labeling rules when transported in small quantities?

  1. Consumer products
  2. Class A Explosives
  3. Poison Gas

6: A driver should count the total number of packages loaded and should count?

  1. The number of damaged packages
  2. The number of packages with reinforcement
  3. Put parking brake on and release air pressure
  4. The number of packages of hazardous materials

7: Poisons must not be transported with which of the following?

  1. Carcinogens
  2. Foods, animal feed or medicines
  3. Fungicides, pesticides or liquid fertilizers

8: What is a way to determine if two materials are not compatible to transport?

  1. Study the charts that are available
  2. Use the trial and error method
  3. Phone the local police or sheriff's office

9: A driver transporting hazardous materials should keep those materials?

  1. Away from other packages
  2. Away from moisture
  3. Away from walls or side racks

10: What is the status of vehicle brakes when hazardous materials are being loaded?

  1. All brakes should be released
  2. The parking brakes should be set
  3. All brakes should be locked

Question Your Answer Correct Answer

Sample Test F

1: Product ID numbers must appear on each side and both ends of a tank containing:

  1. Hazardous materials exceeding 500 gallons
  2. Food products exceeding 1,000 gallons
  3. Hazardous materials exceeding 1,000 gallons
  4. None of the above

2: When transporting explosives you may not:

  1. Park within 5 feet of the traveled part of the road
  2. Within 300 feet of a tunnel
  3. Near an open fire
  4. All of the above

3: Placarded loads require a fire extinguisher with a UL rating of _______ in your truck.

  1. 20 B:C or more
  2. 10 B:C or more
  3. 20 A:C or more
  4. 10 A:C or more

4: When transporting hazardous loads, you must check the tires at least:

  1. Every two hours or 100 miles
  2. Twice every day
  3. Every six hours
  4. Every 200 miles

5: You must carry a gas mask with you when transporting:

  1. Flammable materials
  2. Chlorine
  3. Poison
  4. Liquid Gas

6: If you are required to stop at railroad crossings, you must

  1. Stop 15 to 50 feet before the nearest rail
  2. Remember to never shift gears while crossing tracks
  3. Proceed only when you are sure no train is coming
  4. All of the above

7:  In the event of an accident involving hazardous materials, you should first:

  1. Try to put out any fires
  2. Move leaking materials out of the way
  3. Send someone for help and warn others of the danger
  4. Move your tractor away from the scene

8: A safe haven is defined as:

  1. A designated place in your carrier's parking lot
  2. A government-approved place for parking vehicles loaded with explosives
  3. Any place where someone is watching your tractor and trailer for you.
  4. A shelter in the event of an explosion

9: Which one of the following shipping descriptions is in the proper order:

  1. Phosgene, Poison A, UN1076
  2. Poison A, UN1076, Phosgene
  3. Phosgene, UN1076, Poison A
  4. UN1076, Poison, Phosgene

10: You must have placards for explosives, poison gas or flammables exceeding:

  1. 10 pounds
  2. 10 cartons
  3. 100 pounds
  4. Any amount

Question Your Answer Correct Answer

Sample Test G

1: In order to transport hazardous materials, which require placards:

  1. You must have a commerical driver license
  2. You must pass a written test on hazardous materials
  3. You must have the hazardous materials endorsements on your CDL
  4. All of the above

2: A placard on the trailer is intended to:

  1. Communicate the risk of hazardous materials
  2. Keep people away from the trailer
  3. Stop lumpers from unloading hazardous materials
  4. Remind carriers to give the load to qualified drivers

3: Who is responsible for certifying on the bills of lading that the hazardous materials are prepared properly?

  1. The driver
  2. The shipper
  3. The carrier
  4. The dock workers

4: Containment rules are rules that:

  1. Instruct the driver in loading hazadous materials
  2. Instruct the driver in transporting hazardous materials
  3. Instruct the driver in unloading bulk tanks hauling hazardous materials
  4. All of the above

5: The hazard class of all materials specifies:

  1. How heavy the material is
  2. How much of the material is on your trailer
  3. The risk associated with the materials
  4. How flammable the material is

6: The shipping papers for hazardous materials can be found:

  1. In a pouch on the driver's door
  2. In clear view within reach while driving
  3. On the driver's seat when out of the vehicle
  4. Any of the above locations

7: Special requirements for hazardous materials shipping papers include:

  1. Color coding the hazardous materials bills
  2. Requiring drivers to keep papers either tabbed or on top of other papers
  3. Requiring shippers to keep the bills in a separate envelope
  4. None of the above

8: The shippers must attach a diamond-shaped label to hazardous materials by:

  1. Putting a label on the package
  2. Putting a label on a tag attached to the product
  3. Putting a label on the product
  4. Any one of the above

9: A placarded vehicle must have at least ______ placards?

  1. Two
  2. Three
  3. Four
  4. Six

10: The hazardous materials will be named on:

  1. The Hazardous Materials Table List
  2. The List of Hazardous Substances and Reportable Quantities
  3. Either one of the above lists
  4. All of the above lists

Question Your Answer Correct Answer

Sample Test H

1: Enteries in the Hazardous Materials Table are shown:

  1. In alphabetical order by proper shipping names
  2. In order of risk
  3. In alphabetical order by brand names
  4. In order of quantities being shipped

2: If the hazardous materials are classified as *FORBIDDEN* you should:

  1. Take special caution in transporting the materials
  2. Remove the placards from the trailer
  3. Refuse the load
  4. Place the materials in the front of the trailer

3: The hazardous materials' identification number is important because:

  1. The shipper must include it on the package
  2. Police use the number to identify the materials after an accident
  3. The shipper must include it on the shipping papers
  4. All of the above

4: The designation RQ identifies the material's _____:

  1. Reference Quotient
  2. Regulatory Quotient
  3. Reportable Quantity
  4. Regulatory Quantity

5: You must notify ______ of any spills exceeding the RQ.

  1. Customer
  2. State patrol
  3. DOT and EPA
  4. Federal Highway Administration

6: If the words *INHALATION HAZARD* appear on the shipping papers, you need:

  1. Placards designating a poison
  2. Placards designating a hazardous substance
  3. Face masks in the cab of your truck
  4. Gloves for handling the materials

7: When transporting both hazardous and non-hazardous materials, your bills must:

  1. Describe the hazardous materials first
  2. Have the hazardous materials highlighted
  3. Identify the materials by the letter X or RQ in the HM column
  4. Any one of the above

8: The basic description of a hazardous product must include:

  1. The product's name and identification number
  2. The proper shipping name and the hazard class
  3. The proper shipping name, hazard class and identification number
  4. The shipper's name, the hazard class and a placard

9: When transporting hazardous waste, you must:

  1. Have the word "WASTE" before the name of the material on the bills
  2. Sign a Uniform Hazardous Waste Manifest
  3. Carry a Uniform Hazardous Waste Manifest in the truck
  4. All of the above

10: The placards should be placed on the vehicle:

  1. Before you load it
  2. Before you drive it
  3. Before you get to your destination
  4. Before you cross a scale

Question Your Answer Correct Answer

Passenger and School Bus Endorsement

Passenger Test A

1: How do you certify corrected defects on your pre-trip inspection?

  1. Skip these items on your pre-trip inspection
  2. Sign the previous driver report
  3. Assume that the shop has handled these items

2: During a pre-trip inspection what are some items that you must check?

  1. The standee line
  2. The service brakes - parking brake - steering - lights - reflectors - tires
  3. The number of bandages in first aid kits

3: On which wheels may buses have recapped or regrooved tires?

  1. On all wheels
  2. On all wheels except the front wheels
  3. On all wheels except the drivers

4: What are three items of emergency equipment you must have on a bus?

  1. A fire extinguisher and a first aid kit and tire irons.
  2. Reflectors and stretchers and hydraulic jacks
  3. Reflectors and a fire extinguisher and spare electric fuses

5: Before driving who must inspect the emergency equipment?

  1. The shop
  2. The driver
  3. The dispatcher

6: It is illegal to transport passengers if you have consumed?

  1. An intoxicating beverage within two hours
  2. An intoxicating beverage within four hours
  3. An intoxicating beverage within eight hours.

7: May passengers leave carry-on baggage in a doorway or in the aisle?

  1. Yes, unless there is a printed notice prohibiting it
  2. No, unless no other space is available
  3. No, There should be nothing in a doorway or the aisle that might trip riders

8: What shape are hazardous material labels?

  1. Diamond-shaped
  2. Round.
  3. Rectangular.

9: What is the maximum weight of hazardous materials allowed on a bus?

  1. One hundred pounds.
  2. Five hundred pounds
  3. One thousand pounds

10: What is the total weight of hazardous materials of one class allowed on a bus?

  1. One hundred pounds
  2. Five hundred pounds.
  3. One thousand pounds

Question Your Answer Correct Answer

Passenger Test B

1: Can oxygen be carried on board by a passenger?

  1. No, gases are not permitted on board
  2. Yes, if special written permission is received from the DOT
  3. Yes, if medically prescribed for and in the possession of a passenger

2: Can tear gas be transported on a bus?

  1. No, irritating material may not be carried
  2. Yes, if written permission is received from the DOT
  3. Yes, if the containers meet certain packaging requirements

3: Which of these two items can be carried on a bus by a rider?

  1. Car batteries
  2. Gasoline
  3. Neither one

4: Where must passengers who are standing remain while the bus is underway?

  1. On buses where permitted they must stand at the back of the bus
  2. On buses where permitted they must stand behind the standee line
  3. Passengers are never allowed to stand on buses

5: Can the emergency exit door be opened when the bus is underway?

  1. At a place that is safe for them
  2. The emergency exit door must be open
  3. The emergency exit door will be opened whenever the bus is stopped.

6: If you should discharge an unruly passenger where should this be done?

  1. Within the city limits of an incorporated city
  2. Only at a bus terminal
  3. At a place that is safe for them

7: Where else should you look while driving besides the road ahead?

  1. Also scan the emergency exit door every few seconds
  2. Also scan the emergency equipment to make sure that it is in place
  3. Also scan the interior of the bus.

8: Can looking ahead prevent accidents with cars going in same direction?

  1. You can see changes in the traffic flow early enough to make adjustments
  2. You can see objects that are far away better than those that are close.
  3. You can see where passengers are waiting for your bus in time to stop.

9: If you have to move quickly to avoid an accident you want to know?

  1. Whether all of your passengers are wearing seat belts
  2. Where your passengers are seated.
  3. Where other vehicles are around your bus

10: Which can cause the most dangerous driving condition in poor weather?

  1. Rain.
  2. Ice
  3. Snow

Question Your Answer Correct Answer

Passenger Test C

1: What is a factor that most affects the amount of traction that your bus has?

  1. The width of the road
  2. The type and condition of the road surface
  3. The brand name of your tires

2: In backing up which is more dangerous?

  1. Backing to the left
  2. Backing to the right
  3. They are equally dangerous

3: One rule says that you should have how much following distance?

  1. At least three seconds
  2. At least four seconds
  3. At least five seconds

4: When braking, when will a bus have the most traction?

  1. When the wheels are rolling just short of locking up
  2. During a skid
  3. When the wheels have locked up

5: When driving down a steep hill which is best?

  1. Shift down to a lower gear and not use your brakes.
  2. Shift down to a lower gear so that you will not use your brakes hard.
  3. Use your brakes only

6: If a posted speed is 45 MPH what is a safe speed for your bus?

  1. It may be 45 miles per hour or it could be more.
  2. It will be exactly 45 miles per hour
  3. It may be 45 miles per hour or it could be less

7: Which way will a bus lean if you are driving too fast on a banked curve?

  1. It will lean toward the inside
  2. It will lean in the direction of the turn
  3. It will lean toward the outside.

8: What should you do about speed if the road becomes slippery?

  1. Reduce your speed gradually
  2. Reduce your speed quickly
  3. Stop immediately wherever you are

9: Chains are required to be installed on which wheels?

  1. The front wheels
  2. The drive wheels
  3. The back wheels of a tandem

10: When may a disabled bus be towed to a safe place with passengers on board?

  1. When the emergency exit is not working
  2. Only if getting off the bus would be inconvenient for the passengers
  3. Only if getting off the bus would be unsafe for the passengers

Question Your Answer Correct Answer

School Bus Test A

1: A school bus is "every motor vehicle used for the transportation of children to or from school or school activities."

  1. TRUE
  2. FALSE

2: It doesn't matter if school bus drivers know current laws and regulatons and keep abreast of any changes

  1. TRUE
  2. FALSE

3: School bus drivers must have a commercial driver's license and:

  1. passenger and combination vehicle endorsements
  2. school bus and passenger endorsements
  3. air brake and tank endorsements
  4. none of the above

4: Maximum time in transit: No pupil shall be allowed to remain in transit to and from school for:

  1. more than one hour round trip
  2. more than two hours round trip
  3. more than three hours round trip
  4. more than four hours round trip

5: Can a school bus transport more pupils than indicated by the manufacturer's rated capacity for the bus?

  1. Yes
  2. No

6: Inspection of buses are made one or more times a year in order to determine wherher the school bus can be used to safely transport school children

  1. TRUE
  2. FALSE

7: Each state board of education is charged with the primary responsibility of rules and regulations regarding pupil transportation

  1. TRUE
  2. FALSE

8: A school bus driver doesn't have to wear a seat belt at all times

  1. TRUE
  2. FALSE

9: Only some school bus drivers are required to participate in In-service training programs

  1. TRUE
  2. FALSE

10: Who makes the final decision to determine when weather conditions make the roads unsafe to travel?

  1. The School Administrator
  2. The School Teacher
  3. The School Bus Driver
  4. Any of the above

Question Your Answer Correct Answer

School Bus Test B

1: How far from a railroad crossing should you stop?

  1. Between 10 and 40 feet
  2. Between 15 and 50 feet
  3. Between 20 and 60 feet
  4. Between 25 and 75 feet

2: When must you stop before crossing a drawbridge?

  1. Stop at least 30 feet before the draw of the bridge
  2. Stop at least 40 feet before the draw of the bridge.
  3. Stop at least 50 feet before the draw of the bridge
  4. None of the above

3: What are"prohibited practices" while operating a bus?

  1. Avoid fueling with riders on board
  2. Don't talk to riders while driving
  3. Don't tow or push a disabled bus with riders aboard
  4. All of the above

4: Urban mass transit coaches may have a brake and accelerator interlock shystem. The rear door of a transit bus has to be open to put on the parking brake

  1. TRUE
  2. FALSE

5: If you work as an interstate carrier, you must complete a written inspection report for each bus driver

  1. TRUE
  2. FALSE

6: Are recapped or regrooved tires allowed on buses?

  1. Yes, on all wheels
  2. Yes, on all wheels except the front wheels
  3. No, recapped or regrooved tires are not allowed on buses
  4. As a spare only

7: Before driving who is responsible for inspecting emergency equipment?

  1. The shop
  2. The dispatcher
  3. The driver
  4. All of the above

8: What are the shape of hazardous material labels?

  1. Round
  2. Rectangular
  3. Diamond-shaped
  4. Oval

9: Which poor weather condition can cause the most dangerous driving?

  1. Ice
  2. Rain
  3. Snow
  4. Dark Clouds

10: If you have to swerve quickly to avoid an accident you want to know:

  1. If all of your passengers are wearing seat belts
  2. Where other vehicles are around your bus
  3. Where your passengers are seated
  4. If your emergency flashers work properly

Question Your Answer Correct Answer

School Bus Test C

1: In addition to checking for spare electrical fuses, three red reflective triangles, and a properly charged and rated fire extinguisher, school bus drivers must also inspect the following emergency equipment:

  1. three red burning flares, safety belts in all seats
  2. three red burning flares, a nine-item first-aid kit
  3. three flares of any type and isopryl alcohol.

2: A school bus driver must also check the alternately flashing amber lights indicator, if equipped, the alternately flashing red lights indicator and the strobe light indicator, if equipped

  1. TRUE
  2. FALSE

3: School bus drivers must also check the following external lights and reflectors:

  1. strobe light, if equipped, stop arm light, if equipped
  2. alternately flashing amber lights, if equipped
  3. alternately flashing red lights
  4. all of the above

4: If equipped, check the stop arm to see that it is mounted securely to the left front window of the school bus.

  1. TRUE
  2. FALSE

5: Check that the entry door is not damaged and:

  1. operates smoothly and closes securely from the inside
  2. operates smoothly and closes securely from the outside
  3. operates smoothly and closes securely from a remote location

6: Should the hand rails and step light be checked during a pre-trip inspection?

  1. Yes
  2. No

7: On a pre-trip inspection for a bus, the passenger/entry should be checked for:

  1. door operation, hand and foot rails and handicap lift.
  2. door operation, hand rails, entry step condition and brake condition
  3. door operation, hand rails, entry step condition and handicap lift if equipped

8: When checking a handicap lift the things that you should look for are:

  1. leaking, damaged or missing wheel chairs
  2. leaking, damaged, or missing parts and explain how lift should be checked for correct operation. Lift must be fully retracted and latched
  3. leaking, damaged, or missing parts and explain how lift should be checked for correct operation. Lift must be 25 percent retracted and latched

9: Make sure that all emergency exits are not damaged, operate smoothly and close securely from the outside

  1. TRUE
  2. FALSE

10: Check all emergency exits and show that:

  1. they are not damaged, operate smoothly, and close securely from the inside and the warning devices are working
  2. they are damaged, operate smoothly and close securely and exit warning devices are working.
  3. all devices inside and outside of bus are operating in unison.

Question Your Answer Correct Answer

School Bus Test D

1: In addition to checking for spare electrical fuses, three red reflective triangles, and a properly charged and rated fire extinguisher, school bus drivers must also inspect the following emergency equipment:

  1. three red burning flares, safety belts in all seats.
  2. three red burning flares, a nine-item first-aid kit.
  3. three flares of any type and isopryl alcohol
  4. a cork ball whistle

2: A school bus driver must also check the alternately flashing amber lights indicator, if equipped, the alternately flashing red lights indicator and the strobe light indicator, if equipped.

  1. TRUE
  2. FALSE

3: School bus drivers must also check the following external lights and reflectors:

  1. strobe light, if equipped, stop arm light, if equipped
  2. alternately flashing amber lights, if equipped
  3. alternately flashing red lights
  4. all of the above

4: If equipped, check the stop arm to see that it is mounted securely to the left front window of the school bus.

  1. TRUE
  2. FALSE

5: Check that the entry door is not damaged and:

  1. operates smoothly and closes securely from the inside
  2. operates smoothly and closes securely from the outside
  3. operates smoothly and closes securely from a remote location
  4. the lock is secure

6: Should the hand rails and stop light be checked during a pre-trip inspection?

  1. Yes
  2. No

7: On a pre-trip inspection for a bus, the passenger/entry should be checked for:

  1. door operation, hand and foot rails and handicap lift.
  2. door operation, hand rails, entry step condition and brake condition
  3. door operation, hand rails, entry step condition and handicap lift if equipped.
  4. door operation, hand rails, entry step condition and handicap lift if equipped

8: When checking a handicap lift the things that you should look for are:

  1. leaking, damaged or missing wheel chairs
  2. leaking, damaged, or missing parts and explain how lift should be checked for correct operation. Lift must be fully retracted and latched
  3. leaking, damaged, or missing parts and explain how lift should be checked for correct operation. Lift must be 25 percent retracted and latched
  4. wheel chair chocks

9: Make sure that all emergency exits are not damaged, operate smoothly and close securely from the outside.

  1. TRUE
  2. FALSE

10: Check all emergency exits and show that:

  1. they are not damamged, operate smoothly, and close securely from the inside and the warning devices are working
  2. they are damaged, operate smoothly and close securely and exit warning devices are working
  3. all devices inside and outside of bus are operating in unison
  4. the reflective tape is clean

11: During the pre-trip inspection you should check for broken seat frames and check that seat frames are firmly attached to the floor

  1. TRUE
  2. FALSE

12: Should you check passenger exit mirrors?

  1. Yes
  2. No

13: During the external inspection of a Coach or Transit Bus a check should be made to see that:

  1.  

14: A compartment check includes:

  1. checking that baggage and all other exterior compartment doors are not damaged, operate properly and latch securely.
  2. checking that all compartment doors are painted in coordinating colors
  3. checking that everyone has brought baggage.
  4. all of the above

15: A battery/box check is important. The battery must be secure, connections must be tight and cell caps must be present. In addition these items should be checked:

  1. battery connections should show signs of excessive wear
  2. battery connections should not show signs of excessive corrosion, but the battery box and cover or door is unimportant.
  3. battery connections should not show signs of excessive corrosion and the battery box and cover or door should not be damaged and should be secure
  4. that the mechanic initialed any work performed on the battery

Question Your Answer Correct Answer

School Bus Test E

1: During the pre-trip inspection you should check for broken seat frames and check that seat frames are firmly attached to the floor

  1. TRUE
  2. FALSE

2: Should you check passenger exit mirrors?

  1. Yes
  2. No

3: During the external inspection of a Coach or Transit Bus a check should be made to see that:

  1. the vehicle is sitting level (side to side) and if air-equipped check for water leakage
  2. the vehicle is not sitting level and if air equipped check for audible air leaks from the suspension system
  3. the vehicle is sitting level (front and rear) and if air-equipped, check for audible air leaks from the suspension system
  4. none of the above

4: A compartment check includes:

  1. checking that baggage and all other exterior compartment doors are not damaged, operate properly and latch securely
  2. checking that all compartment doors are painted in coordinating colors
  3. checking that everyone has brought baggage
  4. all of the above

5: A battery/box check is important. The battery must be secure, connections must be tight and cell caps must be present. In addition these items should be checked:

  1. battery connections should show signs of excessive wear
  2. battery connections should not show signs of excessive corrosion, but the battery box and cover or door is unimportant.
  3. battery connections should not show signs of excessive corrosion and the battery box and cover or door should not be damaged and should be secure.
  4. that the mechanic initialed any work performed on the battery

6: What is one of the more dangerous procedures a school bus driver must undertake?

  1. Driving in traffic
  2. Loading and unloading
  3. Driving in the rain
  4. Training replacement drivers

7: When approaching a school bus stop you should activate your overhead amber lights how far from the bus stop?

  1. 100 ft
  2. 200 ft.
  3. 300 ft.
  4. anytime before coming to a stop

8: Immediately after stopping you should:

  1. Open entrance door slighly to activate the stop arms and overhead red warning lights
  2. Tell the children to stand back until you are ready for them to load
  3. Get the children onto the bus as quickly as possible.
  4. Turn the engine off and open the door

9: Always unload on the right outside lane if on a multi-lane road.

  1. TRUE
  2. FALSE

10: Pupils must walk at least how many feet in front of a school bus to cross the road?

  1. 8 ft.
  2. 10 ft
  3. 12 ft
  4. 15 ft

Question Your Answer Correct Answer

School Bus Test F

1: Name some things to check in the interior of a bus during the pre-trip inspection.

  1. Parking brake and steering mechanism
  2. Lights, reflectors, and horn
  3. Tires and windshield wipers
  4. All of the above

2: As you check the outside of the bus, you should close any open emergency exits or open access panels.

  1. TRUE
  2. FALSE

3: When checking the interior of a bus before driving it which of the following parts of the bus must be in sage working condition?

  1. Signaling devices (emergency buzzer)
  2. Handrails, emergency exit handles
  3. Air Conditioners and heaters
  4. A and B

4: Are bus drivers required to wear seatbelts?

  1. Yes
  2. No

5: Is it alright to leave carry on baggage in a doorway or aisle?

  1. Yes
  2. No

6: Most hazardous materials cannot be carried on a bus, but some hazardous materials are allowed

  1. TRUE
  2. FALSE

7: What are some hazardous materials you can transport by bus?

  1. Poison, tear gas, and irritating materials
  2. Small arms ammunition, hospital supplies, and drugs
  3. Explosives and radioactive materials
  4. None of the above

8: In buses designed to allow standing you can stand anywhere you want

  1. TRUE
  2. FALSE

9: What is a standee line?

  1. A strip around the outside of the bus.
  2. A two inch line on the floor to show riders where not to stand
  3. A two inch line between the seats.
  4. Any of the above

10: Does it matter where you make a disruptive passenger get off the bus?

  1. Yes
  2. No

Question Your Answer Correct Answer

School Bus Test G

1: Should the hand rails and stop light be checked during a pre-trip inspection?

  1. Yes
  2. No

2: On a pre-trip inspection for a bus, the passenger/entry should be checked for:

  1. door operation, hand and foot rails and handicap lift.
  2. door operation, hand rails, entry step condition and brake condition
  3. Service couplers
  4. door operation, hand rails, entry step condition and handicap lift if equipped.

3: When checking a handicap lift the things that you should look for are:

  1. leaking, damaged or missing wheel chairs
  2. leaking, damaged, or missing parts and explain how lift should be checked for correct operation. Lift must be fully retracted and latched
  3. leaking, damaged, or missing parts and explain how lift should be checked for correct operation. Lift must be 25 percent retracted and latched

4: Make sure that all emergency exits are not damaged, operate smoothly and close securely from the outside.

  1. TRUE
  2. FALSE

5: Check all emergency exits and show that:

  1. they are not damamged, operate smoothly, and close securely from the inside and the warning devices are working.
  2. they are damaged, operate smoothly and close securely and exit warning devices are working
  3. all devices inside and outside of bus are operating in unison.

6: During the pre-trip inspection you should check for broken seat frames and check that seat frames are firmly attached to the floor

  1. TRUE
  2. FALSE

7: Should you check passenger exit mirrors?

  1. Yes
  2. No

8: During the external inspection of a Coach or Transit Bus a check should be made to see that:

  1. the vehicle is sitting level (side to side) and if air-equipped check for water leakage.
  2. the vehicle is not sitting level and if air equipped check for audible air leaks from the suspension system
  3. the vehicle is sitting level (front and rear) and if air-equipped, check for audible air leaks from the suspension system

9: A compartment check includes:

  1. checking that baggage and all other exterior compartment doors are not damaged, operate properly and latch securely
  2. checking that all compartment doors are painted in coordinating colors
  3. checking that everyone has brought baggage

10: A battery/box check is important. The battery must be secure, connections must be tight and cell caps must be present. In addition these items should be checked:

  1. battery connections should show signs of excessive wear
  2. battery connections should not show signs of excessive corrosion, but the battery box and cover or door is unimportant
  3. battery connections should not show signs of excessive corrosion and the battery box and cover or door should not be damaged and should be secure

Question Your Answer Correct Answer

School Bus Test H

1: During the pre-trip inspection you should check for broken seat frames and check that seat frames are firmly attached to the floor

  1. TRUE
  2. FALSE

2: Should you check passenger exit mirrors?

  1. Yes
  2. No

3: During the external inspection of a Coach or Transit Bus a check should be made to see that:

  1. the vehicle is sitting level (side to side) and if air-equipped check for water leakage
  2. the vehicle is not sitting level and if air equipped check for audible air leaks from the suspension system
  3. the vehicle is sitting level (front and rear) and if air-equipped, check for audible air leaks from the suspension system
  4. none of the above

4: A compartment check includes:

  1. checking that baggage and all other exterior compartment doors are not damaged, operate properly and latch securely
  2. checking that all compartment doors are painted in coordinating colors.
  3. checking that everyone has brought baggage
  4. all of the above

5: A battery/box check is important. The battery must be secure, connections must be tight and cell caps must be present. In addition these items should be checked:

  1. battery connections should show signs of excessive wear
  2. battery connections should not show signs of excessive corrosion, but the battery box and cover or door is unimportant
  3. battery connections should not show signs of excessive corrosion and the battery box and cover or door should not be damaged and should be secure
  4. that the mechanic initialed any work performed on the battery

6: What is one of the more dangerous procedures a school bus driver must undertake?

  1. Driving in traffic
  2. Loading and unloading
  3. Driving in the rain
  4. Training replacement drivers

7: When approaching a school bus stop you should activate your overhead amber lights how far from the bus stop?

  1. 100 ft.
  2. 200 ft
  3. 300 ft.
  4. anytime before coming to a stop

8: Immediately after stopping you should:

  1. Open entrance door slighly to activate the stop arms and overhead red warning lights
  2. Tell the children to stand back until you are ready for them to load.
  3. Get the children onto the bus as quickly as possible.
  4. Turn the engine off and open the door

9: Always unload on the right outside lane if on a multi-lane road.

  1. TRUE
  2. FALSE

10: Pupils must walk at least how many feet in front of a school bus to cross the road?

  1. 8 ft
  2. 10 ft
  3. 12 ft
  4. 15 ft

Question Your Answer Correct Answer
Donate to AITA for providing tests for free.